50
Venue C HAPTER F OUR THE CONCEPT OF venue is similar to personal jurisdiction—the subject of Chap- ter 2—in that it pertains to the geographical area within which a civil action may be brought. However, venue is not con- cerned with the constitutional and statutory limits of a court’s jurisdiction over parties in an action, but with identifying the most sensible and con- venient courts or “venues” within a system in which to litigate a case. In other words, rules of venue function as a means of allocating judicial busi- ness among different courts within the same judicial system. When a case is to be filed in federal court, federal venue rules govern which courts within the federal system are sufficiently related to the parties or to the dispute such that they will serve as a convenient forum in which to litigate the case. Thus, after finding a court that is able to exercise personal jurisdiction over the par- ties in the action (see Chapter 2) and subject matter jurisdiction over the claims raised in the dispute (see Chapter 3), the next step in choosing a proper forum within the federal system is to identify the particular federal district or districts where the case may be brought. * This chapter considers the standards governing venue in the federal system * If a case is being brought within a state court, state venue rules will have to be consulted to identify the court within that state’s judicial system that serves as the proper venue for that case. Here we focus only on the rules governing venue in the federal system. [Ed.] A judicial system refers to the court system within the govern- ment of a particular sovereign. Thus, the federal courts are a separate ju- dicial system from the courts of Vir- ginia, which in turn are a separate judicial system from the courts of Georgia. Venue rules determine the proper court within a single judicial system, not across different systems. FYI FYI FYI FYI FYI FYI Selecting a Court with Jurisdiction Selecting a Convenient Court Applicable Law Pleading & Joinder Discovery Judgment & Appeals Post-Trial Motions Trial Disposition without Trial

Venue - Interactive Casebook Series. Venue rules determine the ... — For all venue purposes — (1) a natural person, ... Report, which publications were distributed to consumers

  • Upload
    lekiet

  • View
    214

  • Download
    1

Embed Size (px)

Citation preview

Venue

chaPter four

ThE concEPT oF venue is similar to personal jurisdiction—the subject of Chap-ter 2—in that it pertains to the geographical area within which a civil action may be brought. However, venue is not con-cerned with the constitutional and statutory limits of a court’s jurisdiction over parties in an action, but with identifying the most sensible and con-venient courts or “venues” within a system in which to litigate a case. In other words, rules of venue function as a means of allocating judicial busi-ness among different courts within the same judicial system. When a case is to be filed in federal court, federal venue rules govern which courts within the federal system are sufficiently related to the parties or to the dispute such that they will serve as a convenient forum in which to litigate the case.

Thus, after finding a court that is able to exercise personal jurisdiction over the par-ties in the action (see Chapter 2) and subject matter jurisdiction over the claims raised in the dispute (see Chapter 3), the next step in choosing a proper forum within the federal system is to identify the particular federal district or districts where the case may be brought.* This chapter considers the standards governing venue in the federal system

* If a case is being brought within a state court, state venue rules will have to be consulted to identify the court within that state’s judicial system that serves as the proper venue for that case. Here we focus only on the rules governing venue in the federal system. [Ed.]

A judicial system refers to the court system within the govern-ment of a particular sovereign. Thus, the federal courts are a separate ju-dicial system from the courts of Vir-ginia, which in turn are a separate judicial system from the courts of Georgia. Venue rules determine the proper court within a single judicial system, not across different systems.

FYIFYI

FYI FYI

FYIFYI

Selectinga Court

withJurisdiction

Selecting aConvenient

Court

ApplicableLaw

Pleading& Joinder

Discovery

Judgment& Appeals

Post-TrialMotions

Trial

Dispositionwithout Trial

Civil Procedure A Contemporary Approach282

as well as the conditions under which an action may be transferred from one venue to another or dismissed altogether when the preferred venue lies outside the federal system.

A. Original Venue

Proper venue in the federal system is determined by various provisions of Title 28 of the U.S. Code. After reviewing these basic provisions and how they are inter-preted and applied generally, we will briefly consider how the venue rules deal with more complex cases involving multiple claims.

1. The Basic Venue Rules

Venue in the federal system is primarily a matter of statutory law. Ordinary cases are governed by the general venue statute, 28 U.S.C. § 1391:

Venue generally: 28 U.S.C. § 1391

(b) Venue in general. — A civil action may be brought in —

(1) a judicial district in which any defendant resides, if all defendants are residents of the State in which the district is located;

(2) a judicial district in which a substantial part of the events or omissions giving rise to the claim occurred, or a substantial part of property that is the subject of the action is situated; or

(3) if there is no district in which an action may otherwise be brought as provided in this section, any judicial district in which any defendant is subject to the court’s personal jurisdiction with respect to such action.

(c) Residency. — For all venue purposes —

(1) a natural person, including an alien lawfully admitted for permanent residence in the United States, shall be deemed to reside in the judicial district in which that person is domiciled;

(2) an entity with the capacity to sue and be sued in its common name under applicable law, whether or not incorporated, shall be deemed to reside, if a defendant, in any judicial district in which such defendant is subject to the court’s personal jurisdiction with respect to the civil action in question * * * and

(3) a defendant not resident in the United States may be sued in any judicial district, and the joinder of such a defendant shall be disregarded in determining where the action may be brought with respect to other defendants.

Chapter 4 Venue 283

Before attempting to parse this provision, let us take a look at a case that illus-trates how courts interpret and apply the general venue statute.

Surface Supplied Inc. v. Kirby Morgan Dive Systems, Inc.

U.S. District Court for the Northern District of California 2013 WL 2355446 (N.D. Cal. May 29, 2013)

MAXINE M. CHESNEY, District Judge.

Before the Court is defendant Kirby Morgan Dive Systems, Inc.’s (“Kirby Mor-gan”) motion, filed April 5, 2013, to dismiss the above-titled action * * *.

BACKGROUND

Plaintiff Surface Supplied Inc. (“Surface Supplied”) is a California corporation, established on June 30, 2011, whose principal place of business is the Northern District of California, and whose three employees reside in said district. [Defendant Kirby Morgan’s “principal facilities” and all but one of its employees are located in the Central District of California.] Surface Supplied “is engaged in the business of research, design, development and manufacture of digital gas analyzer and depth gauge products . . . for the commercial diving industry.” Surface Supplied’s products are “alpha stage products,” and, to date, none have been sold.

Surface Supplied maintains a website “on which it describes the external speci-fications of its products,” but through which it does not “sell or offer [them] for sale.” On the home page of its website, Surface Supplied has used a “cropped version” of “a public domain and well known photograph of an underwater water diver against a backdrop of an American flag.” On three occasions in 2011, Surface Supplied used the cropped version in advertisements appearing in the national publication Under-water Magazine, and on one occasion in the national publication, Marine Technology Report, which publications were distributed to consumers in the Central District [of California]. Surface Supplied also “maintains a presence on the social media sites

(d) Residency of corporations in States with multiple districts. — For purposes of venue under this chapter, in a State which has more than one judicial district and in which a defendant that is a corporation is subject to personal jurisdiction at the time an action is commenced, such corporation shall be deemed to reside in any district in that State within which its contacts would be sufficient to subject it to personal jurisdiction if that district were a separate State, and, if there is no such district, the corporation shall be deemed to reside in the district within which it has the most significant contacts.

Civil Procedure A Contemporary Approach284

Facebook and Twitter” on which it has used a photograph of “a fully outfitted diver standing on seaside rocks.” Surface Supplied’s products are engraved with a logo of a “highly fanciful abstract image of a helmet.” Surface Supplied has also “digitally superimposed this logo over a picture of a tee shirt, which has been displayed on its website and social media sites,” but Surface Supplied “has never printed or distrib-uted any tee shirt bearing this logo.”

On January 22, 2013, Kirby Morgan, a corporation that sells commercial diving helmets and surface gas controllers and analyzers sent Surface Supplied a cease and desist letter, by which letter Kirby Morgan accused Surface Supplied of infringing its trademarks by “using images and representations of Kirby Morgan’s helmets on its website, Facebook page, t-shirts, Twitter and on the panels of Surface Supplied’s gas analyzer equipment” and demanded that Surface Supplied cease its use of the infringing images. The letter also demanded an answer no later than February 1, 2013, and stated, “If Kirby Morgan does not receive a suitable response by the afore-mentioned date, the company will have no choice but to take appropriate legal action which may include immediately seeking a temporary restraining order . . . .”

Thereafter, Surface Supplied requested, and Kirby Morgan granted, a one week extension to February 8, 2013. In lieu of replying, however, Surface Supplied, on February 8, 2013, filed the instant action, seeking a declaration of non-infringement of Kirby Morgan’s federally registered trademarks. Thereafter, on March 15, 2013, Kirby Morgan filed an action in the Central District [of California], alleging, inter alia, federal trademark infringement claims against Surface Supplied. See Kirby Morgan Dive Systems, Inc. v. Surface Supplied, Inc., No. 13–1862 (C.D. Cal. filed Mar. 15, 2013) (“Central District action”).

DISCUSSION

Kirby Morgan moves for dismissal of the instant action pursuant to the Court’s equitable powers under the Declaratory Judgment Act, 28 U.S.C. §§  2201 and 2202 * * *.

Practice Pointer

Notice that Surface Supplied pre-empted the threatened action by Kirby Morgan by taking the initiative and filing a declaratory judgment action first. This permitted Surface Supply to attempt to control the location of the action, which is the subject of the procedural dispute ad-dressed by this opinion. Were Sur-face Supplied’s actions appropriate?

What’s That?

The Declaratory Judgment Act gives district courts discretion to enter-tain cases within their jurisdiction that seek a declaration of legal rights rather than any affirmative relief. Doing so is discretionary, and courts may decline such actions.

???

?

?

?

Chapter 4 Venue 285

I. Motion to Dismiss

“The exercise of jurisdiction under . . . 28 U.S.C. § 2201(a), is committed to the sound discretion of the federal district courts[;][e]ven if the district court has subject matter jurisdiction, it is not required to exercise its authority to hear the

case.” Huth v. Hartford Ins. Co. of the Mid-west, 298 F.3d 800, 802 (9th Cir. 2002). Kirby Morgan argues the instant action is an anticipatory suit that should be dis-missed in light of Kirby Morgan’s later-filed Central District action. In opposi-tion, Surface Supplied argues venue is not proper in the Central District. This Court will not exercise its discretion to dismiss the instant action if the action in the Central District is not properly venued therein, and, consequently, the Court first addresses that issue.

Venue is proper in:

(1) a judicial district in which any defendant resides, if all defendants are residents of the State in which the district is located;

(2) a judicial district in which a substantial part of the events or omissions giving rise to the claim occurred, or a substantial part of property that is the subject of the action is situated; or

(3) if there is no district in which an action may otherwise be brought as provided in this section, any judicial district in which any defendant is subject to the court’s personal jurisdiction with respect to such action.

See 28 U.S.C. § 1391(b). Kirby Morgan argues venue is proper in the Central Dis-trict pursuant to the first two subsections of § 1391(b). The Court addresses each in turn.

A. Judicial District in Which Any Defendant Resides

“A civil action may be brought in . . . a judicial district in which any defendant resides, if all defendants are residents of the State in which the district is located.” See 28 U.S.C. § 1391(b)(1). Surface Supplied is the defendant in the Central District action, and, consequently, for purposes of determining venue for said action, the Court must determine whether Surface Supplied resides in the Central District. “[I]n a State which has more than one judicial district and in which a defendant that is a

Take Note!

Notice how the venue challenge comes into play here: Venue in the Northern District action is not be-ing challenged but, rather, Surface Supplied is challenging venue in the Central District action. Be sure to understand how the question of venue in the Central District action is relevant to the dismissal motion in the Northern District action.

Civil Procedure A Contemporary Approach286

corporation is subject to personal jurisdiction at the time an action is commenced, such corporation shall be deemed to reside in any district in that State within which its contacts would be sufficient to subject it to personal jurisdiction if that district were a separate State . . . .” See 28 U.S.C. § 1391(d). As a “California corporation” with its “principal place of business” in California, Surface Supplied is subject to personal jurisdiction in California, a state having more than one judicial district; consequently, the Court next considers whether Surface Supplied would be subject to personal jurisdiction in the Central District if such district were a separate state.

“Where, as here, there is no applicable federal statute governing personal juris-diction, the district court applies the law of the state in which the district court sits.” See Schwarzenegger v. Fred Martin Motor Co., 374 F.3d 797, 800 (9th Cir. 2004). “Because California’s long-arm jurisdictional statute is coextensive with federal due process requirements, the jurisdictional analyses under state law and federal due process are the same.” See id. at 800–01.

“There are two types of personal jurisdiction, specific and general.” * * * Surface Supplied’s only contacts with the Central District are its advertisements in national magazines, its passive website, and its accounts on Facebook and Twitter [and thus insufficient to support general jurisdiction]. The Court thus turns to specific juris-diction.

The Ninth Circuit has articulated a three-prong test for analyzing specific juris-diction:

(1) The non-resident defendant must purposefully direct his activities or consummate some transaction with the forum or resident thereof; or per-form some act by which he purposefully avails himself of the privilege of conducting activities in the forum, thereby invoking the benefits and protections of its laws;

(2) the claim must be one which arises out of or relates to the defendant’s forum-related activities; and

(3) the exercise of jurisdiction must comport with fair play and substantial justice, i.e. it must be reasonable.

Yahoo! Inc. v. La Ligue Contre Le Racisme Et L’Antisemitisme, 433 F.3d 1199, 1205–06 (9th Cir. 2006) (en banc). Under the first prong, a party “purposefully avails itself of the forum if its contacts with the forum are attributable to (1) intentional acts; (2) expressly aimed at the forum; (3) causing harm, the brunt of which is suffered—and which the defendant knows is likely to be suffered—in the forum.” See Rio Properties, Inc. v. Rio Int’l Interlink, 284 F.3d 1007, 1019 (9th Cir.2002) (citing Calder v. Jones, 465 U.S. 783, 788–89 (1984)).

Chapter 4 Venue 287

The maintenance of a passive website alone does not constitute purposeful availment, see Cybersell, Inc. v. Cybersell, Inc., 130 F.3d 414, 418–20 (9th Cir. 1997); rather, “‘something more’ [is] required to indicate that the defendant purposefully directed its activity in a substantial way to the forum,” see Rio Properties, 284 F.3d at 1020 (quoting Cybersell, 130 F.3d at 418). Here, Kirby Morgan argues Surface Sup-plied’s advertising in Underwater Magazine and Marine Technology Report and its presence on Facebook and Twitter constitute the “something more” required to show purposeful availment.

Advertisements that “specifically target[ ] consumers” in a forum con-stituting a trademark holder’s principal place of business can constitute the req-uisite “something more.” See Rio Prop-erties, Inc., 284 F.3d at 1020 (holding advertisements that “demonstrate[d] an insistent marketing campaign directed toward” forum constituted purpose-ful availment). Advertising in national publications or on Facebook and Twit-ter, however, is not sufficient to support a finding of purposeful availment. See Cascade Corp. v. Hiab–Foco AB, 619 F.2d 36, 37–38 (9th Cir. 1980) (finding no spe-cific jurisdiction where defendant patent holder advertised in “national publications” circulated in forum, visited forum on two occasions, and mailed accusatory letters to plaintiff in forum).

Accordingly, Surface Supplied would not be subject to personal jurisdiction in the Central District were such district a separate state, and, consequently, 28 U.S.C. § 1391(b)(1) does not provide for venue therein.

B. Judicial District in Which a Substantial Part of Events or Omissions Giving Rise to the Claim Occurred

“A civil action may be brought in . . . a judicial district in which a substantial part of the events or omissions giving rise to the claim occurred.” See 28 U.S.C. § 1391(b)(2). The “events or omissions giving rise to the claim[s]” in the Central District action are the design and use of Surface Supplied’s logo and other images that Kirby Morgan alleges infringes its trademarks. As all of Surface Supplied’s activi-ties occurred in the Northern District of California, it cannot be said that a substan-tial part of the events or omissions giving rise to the claims occurred in the Central District. Section 1391(b)(2) thus does not provide for venue in the Central District.

Accordingly, the Court will deny Kirby Morgan’s motion to dismiss.

* * *

_______________

Food for Thought

Recall the discussion of jurisdiction in the Internet context in Chapter 2 and note the requirement for “some-thing more” beyond an Internet pres-ence in a state to support purposeful availment. Would an analysis under Calder v. Jones, 465 U.S. 783 (1984), yield a different conclusion, given the Central District action alleges trademark infringement?

Civil Procedure A Contemporary Approach288

Points for Discussion

a. Three Introductory Points

Before getting into the nuts and bolts of how to conduct an analysis under the general venue statute, it is important to clarify three significant points:

The Possibility of Multiple Proper Venues. First, the venue statute is not designed to give you one venue or the best venue. Rather, several possible venues may come out of an analysis under § 1391. For example, if all the defendants reside in State A, but a substantial part of the events underlying the action happened in State B, then venue would be proper in any federal judicial district within those states where the defendants all reside or where the relevant events occurred. When multiple venues are proper, the plaintiff is free to select among any of the options, constrained only by the need to select a court that could exercise personal jurisdic-tion over all the defendants. This means that to a certain degree, plaintiffs are able to engage in forum shopping, i.e., seeking the most strategically advantageous forum for litigating the action. The plaintiff’s selection, however, is subject to the right of parties to transfer the action to another venue, a topic covered in Section B of this chapter below.

Recent Amendments to the Venue Statutes. Several provisions of the federal venue statutes were amended on December 7, 2011 by the Federal Courts Juris-diction and Venue Clarification Act (JVCA). A principal reform was to eliminate the duplicative provisions of § 1391(a) & (b). The standards formerly set forth in these separate provisions are now found in § 1391(b) alone. Also, what used to appear in § 1391(c) now appears in § 1391(d), with what was in § 1391(d) now appearing in § 1391(c)(3). You will need to make note of these numbering changes when researching cases that predate the JVCA. Minor substantive clarifications were introduced throughout various parts of the venue statutes as well. For a very brief synopsis of the legislation, see the third brAnCh news, Long-Awaited Act Clarifies Venue and Jurisdiction (December 2011).

The Fallback Provisions. Section 1391(b)(3)—which is referred to as the fall-back provision—permits venue in any district having personal jurisdiction over any defendant only if “there is no district in which the action may otherwise be brought.” That means that application of the previous two subdivisions, (b)(1) and (b)(2) have failed to yield any proper venue. This does not mean that venue can be obtained under one of the fallback provisions if an analysis under the first two subdivisions fails to produce a particularly desired venue. If an analysis under the first two sub-divisions identifies any proper venue, even one that fails to match a desired locale, there is no recourse to the fallback provision. See Daniel v. Am. Bd. of Emergency Med., 428 F.3d 408, 434 (2d Cir. 2005) (“[V]enue may be based on

Chapter 4 Venue 289

[(b)(3)] only if venue cannot be established in another district pursuant to any other venue provision.”). In what circumstances might you imagine that the fallback provision would be available for laying venue?

b. Analysis under § 1391

Now, let’s parse the venue statute to clarify how it lays venue in actions filed in federal court. Beginning with subdivision (b)(1), this provision indi-cates that venue is proper in “a judicial district in which any defendant resides, if all defendants are residents of the State in which the district is located.” Thus, in an action by A against B and C, where B resides in the Eastern District of Virginia and C resides in the Western District of Virginia, venue would be proper under (b)(1) in either the Eastern or the Western Districts of Virginia, the state the defendants have in common. Notice that here we are paying no attention to the plaintiff’s state of residence. Why do you think the plaintiff’s residence is irrelevant to a venue analysis under the general venue statute?

What if the defendants reside in a state that has more than one federal judicial district, such as New York? Refer back to the discussion of federal judicial districts in Chapter 1. In states having only one district, the boundaries of the district are coextensive with those of the state, and thus defendants who all reside in such states will be amenable to suit in the district that covers that state, without regard to where within the state they reside. However, in states with multiple judicial districts, venue analysis under § 1391(b)(1) must identify both the defendants’ state of residence and the district within that state in which the defendants reside. This is the analysis featured in the Surface Supplied case above.

Take Note!

Note that a plaintiff must ordinar-ily establish proper venue as to each claim and each defendant. See 14D C. wright & A. miller, Fed. PrAC. & ProC. 3d § 3808.

Go Online

A map showing the current bound-aries of the federal districts can be viewed by visiting http://www.us-courts.gov/uscourts/images/Circuit-Map.pdf.

Hypo 4.1

In an action by X against Y and Z (two individuals), where Y lives in the Southern District of New York and Z lives in the Eastern District of New York, what would be the available proper venue or venues under 1391(b)(1)?

Civil Procedure A Contemporary Approach290

Subdivision (b)(2) provides that venue is proper in “a judicial district in which a substantial part of the events or omissions giving rise to the claim occurred, or a substantial part of property that is the subject of the action is situated.” 28 U.S.C. § 1391(b)(2). How is it determined where a “substantial” portion of the events or omissions underlying the claim occurred? See, e.g., Gulf Ins. Co. v. Glasbrenner, 417 F.3d 353, 357 (2d Cir. 2005) (“[F]or venue to be proper, significant events or omis-sions material to the plaintiff’s claim must have occurred in the district in question, even if other material events occurred elsewhere.”); First of Mich. Corp. v. Bramlet, 141 F.3d 260, 263 (6th Cir. 1998) (holding that “proper venue is not limited to the district where the most substantial event giving rise to the complaint arose” but instead that “the plaintiff may file his complaint in any forum where a sub-stantial part of the events or omissions giving rise to the claim arose”). Should the defendant’s experience of harm be considered an “event” giving rise to the claim for purposes of (b)(2)? See Estate of Abtan v. Blackwater Lodge & Training Ctr., 611 F. Supp. 2d 1, 8 (D.D.C. 2009) (“In tort cases, when determining whether a substantial part of the events or omissions giving rise to the plaintiff’s claim occurred . . . in a particular district for purposes of § 1391(b)(2), . . . courts focus on . . . the place where the allegedly tortious actions occurred and the place where the harms were felt.” (internal quotations and citations omitted)). How was this analysis conducted by the court in Surface Supplied above?

Notice that the events or omissions that are analyzed are limited to those “giv-ing rise to” the claim. What connection should exist between events or omissions and the claim before treating those events or omissions as giving rise to the claim? Should events and omissions only count if they were essential to the existence of the claim under the relevant substantive law? Daniel v. Am. Bd. of Emergency Med., 428 F.3d 408, 433 (2d Cir. 2005) (“When material acts or omissions within the forum bear a close nexus to the claims, they are properly deemed ‘significant’ and, thus, substantial, but when a close nexus is lacking, so too is the substantiality necessary to support venue.”); Jenkins Brick Co. v. Bremer, 321 F.3d 1366, 1371 (11th Cir. 2003) (“Only the events that directly give rise to a claim are relevant. And of the places where the events have taken place, only those locations hosting a ‘substantial part’ of the events are to be considered [for determining venue].”).

c. Determining Residency for Purposes of Venue

Thus far, we have simply asserted that the defendants were residents of a given state. However, an important question is how residency is determined for purposes of venue. Recall that for personal jurisdiction and diversity jurisdiction, the rel-evant consideration for individuals was their state of citizenship, which was in turn determined by domicile. For corporations, state citizenship for purposes of diversity jurisdiction meant their states of incorporation and the state of their principal place of business. Yet another set of rules covered the citizenship of non-corporate col-lective entities such as voluntary associations and partnerships. See Chapter 3. The rules for determining the residency of these types of parties are reviewed below.

Chapter 4 Venue 291

Individuals. The 2011 Jurisdiction and Venue Clarification Act amended § 1391(c) to make clear that an individual’s residency for venue purposes is his or her domicile, something that was not explicit in the previous version of this statute. Recall from our discussion of domicile in Chapter 3 that domicile is the place where a person is physically located and intends to remain for an indefinite period of time. Note also that domicile is purely a concept for individual real persons, not entities.

Corporations. Under 28 U.S.C. § 1391(d) (formerly § 1391(c)), corporations are residents of any district where they are subject to personal jurisdiction. If the state has more than one district, then a corporation is a resident of any district in which it could be subject to personal jurisdiction were that district a state. How does one ana-lyze personal jurisdiction with respect to federal judicial districts? See, e.g., Honor Plastic Indus. Co. v. Lollicup USA, Inc., 2006 WL 2792812, at *5 (E.D. Cal. Sept. 28, 2006) (“Title 28 U.S.C. § 1391 says any corporate defendant ‘shall be deemed to reside in any judicial district in which it is subject to personal jurisdiction at the time the action is commenced.’ This effectively turns the venue question into a per-sonal jurisdiction analysis, treating the Eastern District of California as a state.”). See also QRG, Ltd. v. Nartron Corp., 2006 WL 2583626, at *3 (W.D. Pa. Sept. 7, 2006) (engaging in § 1391(d) analysis). How did the court in Surface Supplied, above, engage in this analysis?

Because corporate residency is tied to personal jurisdiction, multiple possible residencies exist beyond the corporation’s place of incorporation and principal place of business—locales that would confer general jurisdiction over the corporation. All districts within which a corporation would be subject to specific jurisdic-tion for the civil action in question would also be districts in which a cor-poration resides for venue purposes.

Food for Thought

Where can a corporation be said to reside within the state where it is incorporated, when such states have multiple federal districts? Compare Jackson v. Fugro Geoservices, Inc., 2005 WL 3543929, at *1–*2 (E.D. La. 2005) (treating a corporation as a resident of all districts in the state), with Horizon Mktg v. Kingdom Int’l. Ltd., 244 F. Supp. 2d 131, 138 (E.D.N.Y. 2003) (treating a corpora-tion as only resident in the district of its official corporate address within the state), with Tranor v. Brown, 913 F. Supp. 388, 390 (E.D. Pa. 1996) (“[V]enue does not lie against a corporation in a multi-district state solely because of its corporate status. . . . [Section 1391(c)] ensures that corporate defendants cannot be sued in districts within their state of in-corporation with which they have no contacts.” (citations omitted)).

Civil Procedure A Contemporary Approach292

Unincorporated Entities. The Supreme Court has held that for purposes of a venue analysis, the residence of an unincorporated association such as a partnership or trade union is “determined by looking to the residence of the association itself rather than that of its individual members.” Denver & Rio Grande W. R.R. Co. v. Bhd. of R.R. Trainmen, 387 U.S. 556, 559 (1967). The 2011 Jurisdiction and Venue Clarification Act codifies this treatment of unincorporated entities as defendants by

Hypo 4.2

Bernard fell ill after consuming spinach from Williams Farms and sold by Wholesome Markets. Williams Farms is not a corporation but is rather the name under which Pete Williams, a California resident, does business. He ships the spinach grown on his farm to retail outlets throughout the United States including Wholesome Markets. Wholesome Markets is a Delaware corporation with its principal place of business in New York. It operates grocery stores in each of the 50 states. Bernard purchased the spinach that made him sick at the Wholesome Markets store in his home neighborhood in Portland, Oregon.

Bernard initiated an action against Williams and Wholesome Markets for negligence in the U.S. District Court for the District of Oregon. Williams moves to dismiss for improper venue. Should the court grant the motion?

Hypo 4.3

Bernard, an Oregon citizen, fell ill in Texas after consuming spinach from Osage Farms and sold by Wholesome Markets. Osage Farms is a Delaware corporation operating from a farm in Los Angeles, California (the Cen-tral District of California). Osage ships the spinach grown on its farm to retail outlets throughout the United States including Wholesome Markets. Wholesome Markets is a Delaware corporation with its principal place of business in New York. It operates grocery stores in each of the 50 states. Bernard purchased the spinach that made him sick at the Wholesome Markets store in Houston, Texas (Southern District of Texas) during a busi-ness trip. Bernard initiated an action against Osage Farms and Wholesome Markets for negligence in the U.S. District Court for the District of Oregon. Osage moves to dismiss for improper venue. Should the court grant the motion?

Chapter 4 Venue 293

providing that they reside “in any judicial district in which such defendant is sub-ject to the court’s personal jurisdiction with respect to the civil action in question.” 28 U.S.C. § 1391(c)(2). Why do you think this is the case? What justifies look-ing at the association to determine residence for purposes of venue but looking at individual members to determine citizenship for purposes of diversity jurisdiction?

d. Venue in Removed Cases

The language of the general removal statute (see Chapter 3) indicates the federal district in which removed cases must be heard, in effect making those federal dis-tricts the proper venue for removed cases. See 28 U.S.C. § 1441(a); Polizzi v. Cowles Magazines, Inc., 345 U.S. 663, 665–66 (1953) (“The venue of removed actions is governed by 28 U.S.C.A. § 1441(a). . . . Section 1441(a) expressly provides that the proper venue of a removed action is ‘the district court of the United States for the district and division embracing the place where such action is pending.’”). Thus, for removed cases, a separate analysis under the venue statutes is unnecessary and challenges to venue as improper are unavailable, provided the district indicated in § 1441(a) is where the removed case is now being heard. See 28 U.S.C. § 1390(c) (“This chapter shall not determine the district court to which a civil action pending in a State court may be removed. . . .”); Lundahl v. Pub. Storage Mgmt., Inc., 62 Fed. App’x 217, 218–19 (10th Cir. 2003) (“[R]emoval venue is controlled by 28 U.S.C. § 1441(a) which provides that cases may be removed to ‘the district court of the United States for the district and division embracing the place where such action is pending.’ Thus, the dismissal based on § 1391(b) was improvidently granted, and venue in the federal district court for the district of Utah is proper.”); Godfredson v. JBC Legal Grp., P.C., 387 F. Supp. 2d 543, 555 (E.D.N.C. 2005) (“28 U.S.C. § 1391 does not apply when, as here, defendants remove a case to federal court from state court.”).

e. Venue For Alien Defendants

The general venue statute contains a special provision governing the determina-tion of venue in actions against aliens. Section 1391(c)(3)—formerly 1391(d)—states, “a defendant not resident in the United States may be sued in any judicial district . . . .” Thus, if there is a defendant who is not a U.S. citizen, or, presumably, a U.S. citizen domiciled abroad, that defendant may be sued in any federal district court anywhere in the country (as far as venue is concerned; personal jurisdiction and subject matter jurisdiction would be separate matters). This provision also applies to foreign corporations, not just individuals. See Go-Video, Inc. v. Akai Elec. Co., 885 F.2d 1406, 1410–11 (9th Cir. 1989); O.S.C. Corp. v. Toshiba Am., Inc., 491 F.2d 1064, 1068 (9th Cir. 1974) (“[U]nder section 1391(d) as an alien corporation it can be sued in any district.”). If an alien is one of several defendants, the alien is disregarded for purposes of determining venue with respect to the non-alien defen-dants. 28 U.S.C. § 1391(c)(3).

Civil Procedure A Contemporary Approach294

f. Venue and Personal Jurisdiction

Venue and personal jurisdiction are not the same thing. Venue can be proper in a district but that same court can lack personal jurisdiction. See, e.g., World-Wide Volkswagen v. Woodson, 444 U.S. 286 (1980). Alternatively, a court may be able to exercise personal juris-diction over a defendant but venue may be improper there. See Example 4.1.

The distinction between the two concepts does not mean that they will not often coincide, however. Indeed, because corporations reside in any district in which they are subject to personal jurisdiction for purposes of venue, when a court has personal jurisdiction over all the defendants, and all the defendants are corporations, venue will always be proper there because by definition all defendants will “reside” in that juris-diction. See Example 4.2. Of course, when there is only one corporate defendant, venue is proper wherever that defendant could be subjected to personal jurisdiction.

g. Challenging Venue

If a defendant feels that the district in which the plaintiff has brought the action is improper under the applicable venue statute, the proper recourse for challenging venue is a motion to dismiss for improper venue. See Fed. r. Civ. P. 12(b)(3). This is one of the waivable defenses, meaning that under the Federal Rules the motion must be made in a defendant’s initial response to the plaintiff’s complaint or it is waived. See Fed. r. Civ. P. 12(g) & (h). The rules governing the waivable defenses will be covered in greater detail in Chapter 6.

Example 4.1

PFL v. DDE & DNY. All events giving rise to P’s claim arose in California. Under the general venue statute, although DDE would be subject to personal juris-diction in Delaware and DNY would be subject to personal jurisdiction in New York, venue would not be proper in either state but would be proper in the relevant district in California.

Example 4.2

Polly wants to sue ABC, Inc. and XYZ Corp. in California federal court. ABC has substantial operations related to the suit in the Northern District of California and XYZ has the same level of operations in the Southern District of California. Because both defendants are corpora-tions and they would each be subject to personal jurisdiction in the districts where they have related corporate operations, the defendants all reside within the same state (California) and venue would be proper in either the Northern or Southern Districts of California.

Chapter 4 Venue 295

h. Venue and Forum-Selection Clauses

Federal courts must enforce mandatory forum-selection clauses unless the objecting party can “clearly show that enforcement would be unreasonable and unjust, or that the clause [is] invalid for such reasons as fraud or overreaching.” M/S Bremen v. Zapata Off–Shore Co., 407 U.S. 1, 15 (1972). Thus, if there is a valid mandatory forum-selection clause that applies to the dispute before the court, it can provide the basis for proper venue (based on consent) or serve as the basis for a motion to dismiss if the court where the claim has been filed is not the one indicated in the clause. See Magi XXI, Inc. v. Stato della Città del Vaticano, 714 F.3d 714, 720–21 (2d Cir. 2013).

_______________

2. Pendent Venue

Thus far we have considered actions in which venue was proper for each claim under the general venue statue. What happens in cases where there are multiple claims against multiple defendants filed in a particular forum, some of which are properly venued in that forum and others that are not? The next case wrestles with this situation.

PKWare v. Meade

U.S. District Court, Eastern District of Wisconsin 79 F. Supp. 2d 1007 (E.D. Wis. 2000)

ADELMAN, District Judge.

Plaintiff PKWare, Inc. is a Milwaukee company which develops and licenses various software products. Plaintiff is the originator of certain software known as PKZIP software, and owns a patent, trademarks and copyrights on some of the tech-nology related to the software. Defendant Timothy L. Meade is an Ohio resident in the business of translating and reselling software. In September 1992 plaintiff and Meade entered into a contract (the “agreement”) under which Meade would convert some of plaintiff’s software so that it could be used in environments other than those for which it was designed. At the time Meade entered into the agreement he was a sole proprietor doing business under the name “Ascent Solutions.” In 1993 Meade incorporated his business in Ohio under the name Ascent Solutions, Inc. (“ASI”) and became ASI’s majority shareholder, president and CEO.

Civil Procedure A Contemporary Approach296

In 1999 plaintiff commenced this action against both defendants alleging a vari-ety of claims under both state and federal law. These claims include (1) breach of contract, (2) copyright infringement, (3) patent infringement, (4) trademark infringement, (5) false designation of origin, (6)  common law trademark infringement, (7) common law unfair competition, (8) dilution of mark, and (9)  breach of duty of good faith and fair dealing. Pursuant to Fed. r. Civ. P. 12(b)(2) and 12(b)(3), defendants moved to dismiss for lack of personal jurisdiction and improper venue * * * . I now address defendants’ motions.

I. Factual Background

Plaintiff alleges that “on information and belief some time after the effective date of the Agreement, Meade . . . purported to assign his rights and duties under the Agreement to [ASI].” The agreement, however, provides that neither party may assign it without the other party’s written consent. The record contains no evidence that Meade formally

assigned his interest to ASI or that plaintiff consented in writing to such an assign-ment. The parties agree, however, that ASI performed Meade’s responsibilities under the agreement. Further ASI does not dis-pute that it was a party to the agreement despite Meade’s having signed it while he was a sole proprietor. ASI agrees that it had a “business relationship [with] PKWare . . . from September 1992 to the present.”

Under the agreement Meade was to convert or “port” plaintiff’s software so that it could have wider use. Porting is generally accomplished by rewriting a component of the software known as source code. Upon completion of the porting Meade was to send PKWare copies of the resulting software known under the agreement as “Resulting Programs and Software Collections.” Also, under the agreement plaintiff granted Meade a license to resell the converted software in return for a thirty percent royalty which was to be paid monthly. Under the agreement Meade could license the source code to third parties and subcontract the conversion work with plaintiff’s consent but was responsible for the work of subcontractors. Meade’s royalty pay-ments were to be accompanied by monthly reports setting forth the sales for each environment for which the software had been converted during the period that the agreement was in effect. Defendants sent plaintiff some royalty payments and sales reports.

Make the Connection

False designation of origin involves the use of some mark or design in con-nection with a product that gives an erroneous impression of the source of the product. This is a trademark concept that would be covered in an Intellectual Property course.

What’s That?

According to Black’s Law Dictionary, allegations based on information and belief are those “based on second-hand information that the declarant believes to be true.” Rule 11 permits parties to make allegations on this basis.

???

?

?

?

Chapter 4 Venue 297

Meade and plaintiff negotiated the agreement after lengthy communications by telephone, e-mail and writings between Meade in Ohio and plaintiff in Wisconsin. During the course of the agreement Meade communicated with plaintiff’s employ-ees about matters related to the agreement on numerous occasions via telephone and e-mail. Meade also visited Milwaukee once to attempt to hire one of plaintiff’s employees, Steven Burg, during which visit and subsequently “the ongoing relation-ship between PKWare and ASI was occasionally discussed.” The agreement provided that it would be governed by Wisconsin law.

ASI operates an interactive website with an on-line store where users from around the world can place orders for ASI products including some PKZIP products and other software. ASI sells software products all over the world including in Wis-

consin. ASI has also provided products and/or services to at least eighty-six different customers in Wisconsin over the course of the last several years with the majority of these sales occurring in the Eastern District of Wisconsin. ASI also advertises on the internet search engine “AltaVista” and in Computer-World magazine and the SciTech Sci-ence catalogue, all of which have Wisconsin subscribers. * * *

III. Venue

Defendants argue that venue is improper in this judicial district. The burden of showing that venue is proper is on the plaintiff. Grantham v. Challenge-Cook Bros., Inc., 420 F.2d 1182, 1184 (7th Cir. 1969). For venue to lie it must be proper as to both defendants and as to all claims. Payne v. Marketing Showcase, Inc., 602 F. Supp. 656, 658 (N.D. Ill. 1985); Georgene M. Vairo, Determination of Proper Venue, in 17 moore’s FederAl PrACtiCe §  110.05 (Daniel R. Coquillette et al. eds., 3d ed. 1997). Here, plaintiff brings a breach of contract and other state law claims as well as federal claims for trademark, copyright and patent infringement. Because of the variety of plaintiff’s claims, several venue statutes are relevant.

Take Note!

The court here states a general rule: In the face of a defendant’s challenge to venue, it is the plaintiff who has the burden of showing that venue is proper, not the defendant who must show that venue is improper. Why do you think the burden is allocated in this way?

Food for Thought

The court has listed here the defen-dants’ contacts with the forum state, Wisconsin. Based on your under-standing of personal jurisdiction, do you think that the court—which is located in Wisconsin—has personal jurisdiction over the defendants?

Civil Procedure A Contemporary Approach298

A. State Law Claims and Federal Trademark Claims

Venue for plaintiff’s state law claims is governed by the general venue statute, 28 U.S.C. § 1391. The same is true of plaintiff’s federal trademark related claims because the Lanham Act has no special venue provision. Woodke v. Dahm, 70 F.3d 983 (8th Cir. 1995). Section 1391(b) provides that where, as here, subject matter jurisdiction is not founded solely on diversity, venue exists in “(1) a judicial district where any defendant resides, if all defendants reside in the same State, [or] (2) a judicial district in which a substantial part of the events or omissions giving rise to the claim occurred.” ASI can be regarded as residing in this district because, under 28 U.S.C. § 1391(c), a corporation is deemed to reside in a district in which it is subject to personal jurisdiction.* Meade, however, is not a corporation and resides in Ohio. Therefore, venue is not proper as to either defendant under § 1391(b)(1).

However, § 1391(b)(2) provides that venue exists in this district if “a substan-tial part of the events or omissions giving rise to the claim occurred” here. Section 1391(b)(2) is the result of a 1990 amendment which changed the law to the extent that prior law had encouraged an approach that a claim could generally arise in only one venue. Under the amended law, however, venue may be proper in more than one district as long as a “substantial” part of the key events or omissions occurred in the district. In determining where substantial parts of the underlying events occurred I focus on the activities of the defendant and not those of the plaintiff.

In applying the “substantial part” test provided in amended § 1391(b)(2) to breach of contract cases, courts consider a number of factors including where the conduct underlying the breach occurred and where performance under the con-tract was to take place. Consolidated Ins. Co. v. Vanderwoude, 876 F. Supp. 198, 202 (N.D. Ind. 1995). The “substantial part of the events or omissions” standard may be satisfied by a communication transmitted or not transmitted to or from the district in which the cause of action was filed, given a sufficient relationship between the communication and the cause of action. In contract cases courts have held that the delivery or non-delivery of goods and the payment or non-payment of money were significant events providing a basis for venue in the district where they were to occur. See American Carpet Mills v. Gunny Corp., 649 F.2d 1056, 1059 (5th Cir. 1981) (place where goods were to be delivered was place of performance); Gardner Eng’g Corp. v. Page Eng’g Co., 484 F.2d 27, 33 (8th Cir. 1973) (venue existed at site where delivery was to be made); Decker Coal Co. v. Commonw. Edison Co., 805 F.2d 834, 843–44 (9th Cir. 1986) (venue for claim based on breach of contract is in place of intended performance rather than place of repudiation); Oce-Industries, Inc. v. Coleman, 487 F. Supp. 548, 552 (N.D. Ill. 1980) (place where orders accepted and payment to be made important for venue purposes).

* In an omitted portion of the opinion, the court concluded that personal jurisdiction was proper with respect to ASI in Wisconsin. [Ed.]

Chapter 4 Venue 299

In the present case defendants were obliged under the agreement to deliver copies of software, make royalty payments and send sales reports to plaintiff in this district and allegedly failed to do so. Defendants were also supposed to obtain approval for sublicensing and distributing in this district and allegedly failed to do so. Further, defendants allegedly attempted unsuccessfully to cure their default here. These alleged failures constitute a substantial part of the events or omissions giving rise to plaintiff’s claim. Thus, based on the facts of the case and the foregoing authority, it is reasonable to conclude that venue exists in this district with respect to plaintiff’s state law and trademark claims.

B. Federal Copyright Claims

Title 28 U.S.C. § 1400(a) governs venue in copyright claims and pro-vides that venue is proper in a district in which the defendant “resides” or “may be found.” Under this provi-sion, a defendant “may be found” in a district if the defendant is subject to personal jurisdiction in that forum. Because I have found that defendants are subject to personal jurisdiction here venue is proper under 28 U.S.C. §  1400(a) with respect to plaintiff’s copyright claims.

C. Patent Infringement Claim

Title 28 U.S.C. § 1400(b) governs plaintiff’s patent infringement claim. It pro-vides that an action may be brought in a district “where the defendant resides or where the defendant has committed acts of infringement and has a regular and established place of business.”

1. Claim Against ASI

It is not disputed that ASI has no “regular and established place of business” in this district. The question, therefore, is whether for § 1400(b) purposes ASI “resides” in the Eastern District of Wisconsin. ASI argues that it resides in Ohio where it is incorporated. Title 28 U.S.C. § 1391(c), however, provides that “[f]or purposes of venue under this chapter, a defendant that is a corporation shall be deemed to reside in a judicial district in which it is subject to personal jurisdiction” (emphasis added). Read in conjunction with § 1400(b)—because § 1400(b) is also part of chapter 87 of title 28—§ 1391(c) expands the definition of “resides.” Braden Shielding Sys. v.

Take Note!

As mentioned earlier, there are special venue statutes that govern certain types of cases. When these statutes apply, they indicate proper venues for their respective claims. Thus, the court here applies the spe-cial statute governing venue in fed-eral copyright claims and the patent action venue provision to the patent infringement claim below.

Civil Procedure A Contemporary Approach300

Shielding Dynamics, 812 F.Supp. 819, 820 (N.D. Ill. 1992). Thus, it would appear that for purposes of §1400(b), ASI resides in this district. ASI, however, contests this proposition and argues that Fourco Glass Co. v. Transmirra Products Corp., 353 U.S. 222 (1957), provides that § 1400(b) by itself governs venue in patent infringement cases.

The identical issue was addressed in Braden, which I believe analyzed it correctly. Braden began by pointing out that district courts are bound by the decisions of the federal circuit in patent cases and that the federal circuit had recently addressed the issue in VE Holding Corp. v. Johnson Gas Appliance Co., 917 F.2d 1574, 1578 (Fed. Cir. 1990). The VE Holding court found that amended §  1391(c) “re-defines the meaning of the term ‘resides’” in § 1400(b) for cor-porate defendants. Braden then went on to show how the conflict between Fourco and VE Holding was more appar-ent than real and that both cases used the same analytic framework:

The amendments to § 1391 required the court in VE Holding to decide whether Congress sought to change the established substance of § 1400(b) by requiring the specific patent venue provision to be read in conjunction with the general venue provision. The court emphasized two things: (1) the statutory language added in 1988 to § 1391(c), “[f]or purposes of venue under this chapter,” referred clearly to §§  1391–1412 (and, necessarily, § 1400(b)), and thus amounted to the “exact and classic language of in-corporation” that was missing in the 1948 version of § 1391(c) construed in Fourco; and (2) the drafters of the amendment were well aware of the issue of incorporation and so the obvious meaning of the words they chose should be respected.

Braden, 812 F. Supp. at 821.

Thus, for purposes of venue, ASI “resides” in this district if it was subject to personal jurisdiction here when the action was commenced. As I have already deter-mined that ASI is subject to personal jurisdiction, venue in this district for plaintiff’s patent infringement claim is proper.

2. Meade

Section 1400(b) also governs venue with respect to plaintiff’s patent infringe-ment claim against Meade. Meade is not a corporation and resides in Ohio. Venue,

The Federal Circuit is the federal appellate court that has juris-diction over appeals involving patent matters. This means that such cases in the federal district courts through-out the country must be appealed to this court rather than the relevant regional circuit court in which those district courts are located. Visit www.cafc.uscourts.gov/ for more informa-tion.

FYIFYI

FYI FYI

FYIFYI

Chapter 4 Venue 301

therefore, is not proper here under the “resides” clause of § 1400(b) and will only lie under § 1400(b) if Meade “committed acts of infringement” and has a “regular and established place of business” in this district. “Having a ‘regular and established place of business’ involves more than ‘doing business.’” Brunswick Corp. v. Suzuki Motor Co., 575 F. Supp. 1412, 1423 (E.D. Wis. 1983). Meade does not have a regular and estab-lished place of business in this judicial district. He neither owns nor rents property, nor employs sales agents here. Therefore, venue is not proper in this district under § 1400(b) with respect to the patent infringement claim against Meade.

Plaintiff, however, asks the court to exercise pendent venue over the improperly venued claim. Meade objects pointing out both that patent infringement claims are governed exclusively by § 1400(b) and that venue must generally be established for each separate cause of action. Courts have sometimes relied on a theory of pen-dent venue analogous to the doctrine of supplemental jurisdiction. See, e.g., Travis v. Anthes Imperial Ltd., 473 F.2d 515, 528–29 (8th Cir. 1973) (when venue is proper on federal claims, it is also proper on pendent state claims); Beattie v. United States, 756 F.2d 91, 101 (D.C. Cir. 1984) (the doctrine of “pendent venue” is now well established). Generally, pendent venue will be applied if a court may also exercise supplemental jurisdiction.

However, there is no practical reason to limit the application of pendent venue to pendent jurisdiction cases, and judges have recognized this by utilizing pendent venue in other situations. In Zenith Radio Corp. v. Matsushita Elec. Ind. Co., 402 F. Supp. 262 (E.D. Pa. 1975), plaintiff brought claims under two different federal statutes, one of which was properly venued and one of which was not. The court retained jurisdic-tion of both, holding that it would be senseless to bifurcate the case by dismissing or transferring some counts while retaining the others, since the facts underlying both claims were essentially the same. See also Seamon v. Upham, 563 F. Supp. 396, 399 (E.D. Tex. 1983), where the court noted that pendent venue could be exercised if it furthered the goals of judicial economy, convenience and fairness to the litigants.

However, courts have been less willing to accept venue of a claim for patent infringement based on the doctrine of pendent venue. Patent infringement cases are governed by a specific venue statute, § 1400(b), and courts have expressed the view that application of the doctrine of pendent venue is inconsistent with the specific requirements of the statute. See Bradford Novelty Co. v. Manheim, 156 F. Supp. 489, 491 (S.D.N.Y. 1957) (Congress saw fit to narrowly confine the venue provisions applicable to this type of action). The Supreme Court’s comments about § 1400(b) also counsel caution in regard to applying pendent venue to patent infringement cases. See Schnell v. Peter Eckrich & Sons, 365 U.S. 260, 264 (1961) (“The require-ment of venue is specific and unambiguous; it is not one of those vague principles which, in the interest of some overriding policy, is to be given a liberal construc-tion.”). Additionally, no Seventh Circuit case authorizes the application of pendent venue in this case.

Civil Procedure A Contemporary Approach302

Thus, I decline to apply the doctrine of pendent venue to plaintiff’s patent infringement claim against Meade. Therefore, venue with respect to the claim is improper in this district. Pursuant to Fed. r. Civ. P. 12(b)(3), Meade has moved to dismiss this claim. His motion will be granted.

* * *

_______________

Points for Discussion

a. Pendent Venue

When a plaintiff asserts multiple claims in an action, venue must ordinarily be proper for each of those claims. Centerville ALF, Inc. v. Balanced Care Corp., 197 F. Supp. 2d 1039, 1046 (S.D. Ohio 2002) (“Plaintiffs bear the burden of establishing that venue is proper . . . and must demonstrate that venue is proper for each claim asserted in their complaint.”). However, many courts have recognized an exception to this general rule in the doctrine of pendent venue, “according to which venue may be proper with respect to an improperly venued claim if the claim is joined to a properly venued claim, and the claims arise out of a common nucleus of operative fact.” 14D C. wright, A. miller & e. CooPer, Fed. PrAC. & ProC. Juris. 3d § 3808. See also High River Ltd. P’ship v. Mylan Lab., Inc., 353 F. Supp. 2d 487, 493 (M.D. Pa. 2005) (“Pendent venue is an exception to the rule that venue must be established for each cause of action asserted in the complaint.”). But see McCaskey v. Cont’l Airlines, Inc., 133 F. Supp. 2d 514, 525 (S.D. Tex. 2001) (“Simply put, there is no pendant [sic] venue under 28 U.S.C. § 1391(a)(2).”).

What do you think is the rationale for this judicially-recognized exception? Is the notion of pendent venue consistent with the general venue statute? With respect to claims governed by specialized venue statutes, should the doctrine of pendent venue be available to lay venue in forums not approved by those statutes? See, e.g., Price v. Countrywide Home Loans, Inc., 2005 WL 2354348, at *5–*6 (S.D. Ga. Sept. 26, 2005).

b. Venue for Subsequently Joined Claims

How should courts assess venue for claims that are asserted against the original parties by way of counterclaims, crossclaims, or by outsiders seeking to intervene in the case? Generally speaking, courts have not required an independent assessment of venue for such additional claims. See 14D C. wright, A. miller & e. CooPer, Fed. PrAC. & ProC. Juris. 3d § 3808 (“When these additional claims are asserted against parties to the original suit, no venue objection should be available. Since the main action satisfied the venue statutes, these additional claims asserted later need not.”).

Chapter 4 Venue 303

c. Special Venue Statutes

The general venue statute, by its own terms, applies “except otherwise provided by law.” As illustrated in PKWare, there are indeed other venue provisions for specific types of cases that might trump the general venue provisions of § 1391 when they apply. See, e.g., 28 U.S.C. § 1400 (copyright and patent cases). Thus, in assessing venue it is important first to determine whether one of these special statutes applies before moving on to an analysis of venue under the general venue statute. Note, however, that the alien venue provision, § 1391(c)(3) (formerly § 1391(d)), itself trumps special venue statutes. See Brunette Mach. Works, Ltd. v. Kockum Indus., Inc., 406 U.S. 706, 714 (1972) (holding that § 1391(d) is “a declaration of the long-estab-lished rule that suits against aliens are wholly outside the operation of all the federal venue laws, general and special”). Courts have also held that when the general venue statute and the special venue statute are not in conflict, and the special venue statute is not exclusive, litigants may be able to lay venue by satisfying either standard that applies. See 14D C. wright, A. miller & e. CooPer, Fed. PrAC. & ProC. Juris. 3d § 3818 (“[T]he Supreme Court has now held that special venue statutes are supple-mented by, and are to be read in the light of, the liberalizing provisions of the general venue statutes, at least in the absence of contrary restrictive indications in the special venue statutes.” (citing Pure Oil Co. v. Suarez, 384 U.S. 202 (1966))).

_______________

B. Change of Venue

What happens if the court where the plaintiff files its case is not a proper venue under the applicable venue statute? Or, what recourse does a defendant have if she simply finds the selected venue to be too inconvenient, even though venue may be proper? Taking the latter situation first, federal law provides courts with the power to transfer cases to another federal judicial district—even though venue in the first district may be proper—on the ground that doing so is “in the interest of justice” or better for the “convenience of parties and witnesses”:

Change of Venue: 28 U.S.C. § 1404(a)

(a) For the convenience of parties and witnesses, in the interest of justice, a district court may transfer any civil action to any other district or division where it might have been brought or to any district or division to which all parties have consented.

Civil Procedure A Contemporary Approach304

Thus, a defendant who feels that the plaintiff’s chosen venue is too inconvenient or otherwise unduly burdensome may seek a transfer of the case to another federal judicial district (or division within the initially chosen district).

On the other hand, if the plain-tiff initially files an action in a forum that is not a proper venue under the applicable venue statute, then the defendant may still seek to transfer the case but pursuant to a separate venue transfer provision:

In this section, we will cover two issues presented by these venue transfer provi-sions: (1) To which federal districts may an action be transferred, and (2) What standards govern transfer under § 1404 and § 1406?

1. Determining Available Transferee Courts

Hoffman v. Blaski*

Supreme Court of the United States 363 U.S. 335 (1960)

Mr. Justice WHITTAKER delivered the opinion of the Court.

* * *

The instant cases present the question whether a District Court, in which a civil action has been properly brought, is empowered by § 1404(a) to transfer the action, on the motion of the defendant, to a district in which the plaintiff did not have a right to bring it.

* This case was resolved along with a companion case, Sullivan v. Behimer, which raised the same question as Hoffman v. Blaski. Discussion of the facts of Sullivan is omitted here. [Ed.]

Take Note!

Both § 1404 and § 1406 below per-mit transfer to a “district.” A district refers to one of the federal judicial districts discussed above. Thus, un-der the federal venue transfer stat-utes, an action may not be transferred from federal court to a state court.

Curing Defective Venue: 28 U.S.C. § 1406(a)

(a) The district court of a district in which is filed a case laying venue in the wrong division or district shall dismiss, or if it be in the interest of justice, transfer such case to any district or division in which it could have been brought.

Chapter 4 Venue 305

Respondents, Blaski and others, residents of Illinois, brought this patent infringement action in the United States District Court for the Northern District of Texas against one Howell and a Texas corporation controlled by him, alleging that the defendants are residents of, and maintain their only place of business in, the City of Dallas, in the Northern District of Texas, where they are infringing respondents’ patents. After being served with process and filing their answer, the defendants moved, under § 1404(a), to transfer the action to the United States District Court for the Northern District of Illinois.2 Respondents objected to the transfer on the ground that, inasmuch as the defendants did not reside, maintain a place of busi-ness, or infringe the patents in, and could not have been served with process in, the Illinois district, the courts of that district lacked venue over the action3 and ability to command jurisdiction over the defendants; that therefore that district was not a forum in which the respondents had a right to bring the action, and, hence, the court was without power to transfer it to that district. Without mentioning that objection or the question it raised, the District Court found that “the motion should be granted for the convenience of the parties and witnesses in the interest of justice,” and ordered the case transferred to the Illinois district. Thereupon, respondents moved in the Fifth Circuit for leave to file a petition for a writ of mandamus direct-ing the vacation of that order. That court, holding that “(t)he purposes for which § 1404(a) was enacted would be unduly circumscribed if a transfer could not be made ‘in the interest of justice’ to a district where the defendants not only waive venue but to which they seek the transfer,” denied the motion.

Upon receipt of a certified copy of the pleadings and record, the Illinois District Court assigned the action to Judge Hoffman’s calendar. Respondents promptly moved for an order remanding the action on the ground that the Texas District Court did not have power to make the transfer order and, hence, the Illinois District Court was not thereby vested with jurisdiction of the action. After expressing his view that the “weight of reason and logic” favored “retransfer of this case to Texas,” Judge Hoffman, with misgivings, denied the motion. Respondents then filed in the Seventh Circuit a petition for a writ of mandamus directing Judge Hoffman to reverse his order. After hearing and rehearing, the Seventh Circuit, holding that “(w)hen Congress provided (in § 1404(a)) for transfer (of a civil action) to a district ‘where it might have been brought,’ it is hardly open to doubt but that it referred to a district where the plaintiff * * * had a right to bring the case,” and that respondents did not have a right to bring this action in the Illinois district, granted the writ, one judge dissenting.

* * *

2 The asserted basis of the motion was that trial of the action in the Illinois District Court would be more convenient to the parties and witnesses and in the interest of justice because several actions involving the validity of these patents were then pending in that court, and that pretrial and discovery steps taken in those actions had developed a substantial amount of evidence that would be relevant and useful in this action. Defendants also stated in the motion that, if and when the case be so transferred, they would waive all objections to the venue of the Illinois District Court over the action and would enter their appearance in the action in that court.3 See 28 U.S.C. § 1400(b).

Civil Procedure A Contemporary Approach306

Petitioners’ “thesis” and sole claim is that § 1404(a), being remedial, should be broadly construed, and, when so construed, the phrase “where it might have been brought” should be held to relate not only to the time of the bringing of the action, but also to the time of the transfer; and that “if at such time the transferee forum has the power to adjudicate the issues of the action, it is a forum in which the action might then have been brought.” (Emphasis added.) They argue that in the interim between the bringing of the action and the filing of a motion to transfer it, the defen-dants may move their residence to, or, if corporations, may begin the transaction of business in, some other district, and, if such is done, the phrase “where it might have been brought” should be construed to empower the District Court to transfer the action, on motion of the defendants, to such other district; and that, similarly, if, as here, the defendants move to transfer the action to some other district and consent to submit to the jurisdiction of such other district, the latter district should be held one “in which the action might then have been brought.” (Emphasis added.)

We do not agree. We do not think the § 1404(a) phrase “where it might have been brought” can be interpreted to mean, as petitioners’ theory would required, “where it may now be rebrought, with defendants’ consent.” This Court has said, in a different context, that § 1404(a) is “unambiguous, direct [and] clear,” Ex parte Collett, 337 U.S. 55, 58 (1949), and that “the unequivocal words of § 1404(a) and the legislative history * * * (establish) that Congress indeed meant what it said.” United States v. National City Lines, Inc., 337 U.S. 78, 84 (1949). * * * [W]e think the dissenting opinion of Judges Hastie and McLaughlin in Paramount Pictures, Inc. v. Rodney, 186 F.2d 111, 119 (3d Cir. 1951), correctly answered this contention:

But we do not see how the conduct of a defendant after suit has been instituted can add to the forums where ‘it might have been brought.’ In the normal meaning of words this language of Section 1404(a) directs the attention of the judge who is considering a transfer to the situation which existed when suit was instituted.

It is not to be doubted that the transferee courts, like every District Court, had jurisdiction to entertain actions of the character involved, but it is obvious that they did not acquire jurisdiction over these particular actions when they were brought in the transferor courts. The transferee courts could have acquired jurisdiction over these actions only if properly brought in those courts, or if validly transferred thereto under § 1404(a). Of course, venue, like jurisdiction over the person, may be waived. A defendant, properly served with process by a court having subject matter jurisdiction, waives venue by failing seasonably to assert it, or even simply by mak-ing default. But the power of a District Court under § 1404(a) to transfer an action to another district is made to depend not upon the wish or waiver of the defendant but, rather, upon whether the transferee district was one in which the action “might have been brought” by the plaintiff.

Chapter 4 Venue 307

The thesis urged by petitioners would not only do violence to the plain words of § 1404(a), but would also inject gross discrimination. That thesis, if adopted, would empower a District Court, upon a finding of convenience, to transfer an action to any district desired by the defendants and in which they were willing to waive their statutory defenses as to venue and jurisdiction over their persons, regardless of the fact that such transferee district was not one in which the action “might have been brought” by the plaintiff. Conversely, that thesis would not permit the court, upon motion of the plaintiffs and a like showing of convenience, to transfer the action to the same district, without the consent and waiver of venue and personal jurisdiction defenses by the defendants. Nothing in § 1404(a), or in its legislative history, suggests such a unilateral objective and we should not, under the guise of interpretation, ascribe to Congress any such discriminatory purpose.

* * *

Inasmuch as the respondents (plaintiffs) did not have a right to bring these actions in the respective transferee districts, it follows that the judgments of the Court of Appeals were correct and must be affirmed.

Affirmed.

[A concurrence by Justice STEWART and dissents by Justices FRANKFURTER, HARLAN, and BRENNAN have been omitted.]

_______________

Points for Discussion

a. Proper Transferee Courts under § 1404 & § 1406

Under Hoffman, only those dis-tricts where the plaintiff could have initially filed the action properly, with-out consent of the defendant, are per-missible transferee courts for purposes of §  1404. However, the 2011 Juris-diction and Venue Clarification Act (JVCA) amended § 1404(a) to permit a transfer “to any district or division to which all parties have consented.” Does this language overrule Hoffman completely? If not, what is the continuing relevance of the case, if any? Under what circumstances would all parties, including

The JVCA also amended § 1404 to bar transferring cases to the District Courts for Guam, the North-ern Mariana Islands, and the Virgin Islands. 28 U.S.C. §  1404(d). A similar amendment was not made to § 1406.

FYIFYI

FYI FYI

FYIFYI

Civil Procedure A Contemporary Approach308

the plaintiff, have consented to litigating in a district? See, e.g., Atl. Marine Constr. Co. v. U.S. Dist. Court for W. Dist. of Tex., 134 S. Ct. 568, 579 (2013) [FLEX Case® 4.A], (noting that the jurisdiction identified in a forum-selection clause would be a district to which all parties have consented under 1404(a)). To which districts may a case be transferred under § 1406, which limits transferee districts to those where the case “could have been brought”?

b. Forum-Selection Clauses

Parties to contracts are free to identify the forum in which any disputes that arise under the contract will be litigated. If a plaintiff initiates an action in a federal judicial district different from the one identified in a governing forum-selection clause, the defendant may seek to enforce the forum-selection clause by a motion to transfer under § 1404(a). See Atl. Marine, 134 S. Ct. at 579 (“Although a forum-selection clause does not render venue in a court ‘wrong’ or ‘improper’ within the meaning of § 1406(a) or Rule 12(b)(3), the clause may be enforced through a motion to transfer under § 1404(a).”). Indeed, the Supreme Court recently held that when forum-selection clauses are the basis for a § 1404 transfer motion, “a district court

should transfer the case unless extraordi-nary circumstances unrelated to the convenience of the parties clearly disfa-vor a transfer.” Id. at 575. However, if the forum-selection clause points to a state or foreign court as the agreed forum for a dispute, the appropriate way to enforce such a clause would be through the doctrine of forum non conveniens, see id. at 580, a topic covered later in this chapter.

c. Transfers in the Absence of Personal Jurisdiction

It is quite possible that a plaintiff will file an action in a federal district court that is an improper venue and lacks personal jurisdiction over the defendants. Should the district court be able to transfer the case even though it lacks jurisdiction over the defendants, or should the court only be able to dismiss the action for lack of personal jurisdiction and permit the plaintiff to refile in another court? See Goldlawr, Inc. v. Heiman, 369 U.S. 463, 466 (1962) (“The language of §  1406(a) is amply broad enough to authorize the transfer of cases, however wrong the plaintiff may have been in filing his case as to venue, whether the court in which it was filed had personal jurisdiction over the defendants or not.”).

Take Note!

The Supreme Court in Atlantic Marine expressly did not consider the ques-tion of whether a defendant can seek dismissal under Rule 12(b)(6) if the plaintiff files suit in a district other than the one specified in a valid fo-rum-selection clause.

Chapter 4 Venue 309

Alternatively, when the initial venue is improper, what if the prospective trans-feree court has proper venue but would lack personal jurisdiction over the defen-dant? In that case, per Hoffman, the court would not be one “where it might have been brought” and thus the case could not be transferred there. Is the transferor court required to determine the amenability of the defendant to personal jurisdic-tion before granting a transfer? Which party should bear the burden on this issue? See Mellor v. Moe, 2006 WL 1644877, at *1 n.2 (D.N.J. June 14, 2006) (noting that “a finding of personal jurisdiction over [the defendant] in the proposed transferee court is a prerequisite to transfer pursuant to § 1406(a),” and that the moving party must present proof that personal jurisdiction is proper in the transferee court).

d. Multidistrict Litigation

There is a special venue provision that enables cases to be transferred outside of § 1404 or § 1406 to any federal district court selected by a special panel of judges who are appointed to oversee such transfers. Under 28 U.S.C. § 1407, civil actions sharing common questions of fact pending in different federal districts may be trans-ferred to any district for consolidated pretrial proceedings. Such transfers are deter-mined and implemented by a special body called the Judicial Panel on Mul-tidistrict Litigation (JPML). If there is a trial in such cases, however, they are to be remanded to their respective original transferor courts. See Lexecon Inc. v. Milberg Weiss Bershad Hynes & Lerach, 523 U.S. 26 (1998).

2. Standards for Venue Transfers

Having reviewed the methods of identifying proper transferee courts, we now turn to a consideration of the standards that transferor courts use to determine whether a case should be transferred. These standards are illustrated in the two cases that follow.

Go Online

Students interested in learning more about the JPML can visit its website at www.jpml.uscourts.gov.

Civil Procedure A Contemporary Approach310

Smith v. Colonial Penn Insurance Co.

U.S. District Court for the Southern District of Texas 943 F. Supp. 782 (S.D. Tex. 1996)

KENT, District Judge.

This is a breach of contract case based on an insurance contract entered into by Plaintiff and Defendant. Now before the Court is Defendant’s October 11, 1996 Motion to Transfer Venue from the Galveston Division to the Houston Division of the United States District Court for the Southern District of Texas pursuant to 28 U.S.C. §  1404(a). For the reasons set forth below, the Motion is DENIED.

Section 1404(a) provides: “For the convenience of parties and witnesses, in the interest of justice, a district court may transfer any civil action to any other district or division where it might have been brought.” 28 U.S.C. § 1404(a). The defendant bears the burden of demonstrating to the District Court that it should, in its sound

discretion, decide to transfer the action. The Court weighs the follow-ing factors to decide whether a transfer is warranted: the availability and con-venience of witnesses and parties, the location of counsel, the location of books and records, the cost of obtain-ing attendance of witnesses and other trial expenses, the place of the alleged wrong, the possibility of delay and prejudice if transfer is granted, and the plaintiff’s choice of forum, which is generally entitled to great deference.

Defendant’s request for a transfer of venue is centered around the fact that Galves-ton does not have a commercial airport into which Defendant’s employees and corpo-rate representatives may fly and out of which they may be expediently whisked to the federal courthouse in Galveston. Rather, Defendant contends that it will be faced with the huge “inconvenience” of flying into Houston and driving less than forty miles to the Galveston courthouse, an act that will “encumber” it with “unnecessary driving time and expenses.” The Court certainly does not wish to encumber any litigant with such an onerous burden. The Court, being somewhat familiar with the Northeast,

Judge Kent, the author of this opinion, was impeached by the U.S. House of Representatives after being criminally charged for sexually harassing his employees. He resigned rather than face conviction and re-moval from office by the Senate and subsequently was sentenced to 33 months imprisonment for obstruct-ing the criminal investigation into his sexual misconduct. See en.wikipedia.org/wiki/Samuel_B._Kent.

FYIFYI

FYI FYI

FYIFYI

Take Note!

In this paragraph, the court sets forth several factors that are relevant to a determination of whether a transfer is warranted. Make a note of these fac-tors because they can help give sub-stance to your analysis of whether a transfer would be “in the interest of justice” or promote the convenience of the parties and witnesses.

Chapter 4 Venue 311

notes that perceptions about travel are different in that part of the country than they are in Texas. A litigant in that part of the country could cross several states in a few hours and might be shocked at having to travel fifty miles to try a case, but in this vast state of Texas, such a travel distance would not be viewed with any surprise or consternation. Defendant should be assured that it is not embark-ing on a three-week-long trip via cov-ered wagons when it travels to Galves-ton. Rather, Defendant will be pleased to discover that the highway is paved and lighted all the way to Galveston, and thanks to the efforts of this Court’s predecessor, Judge Roy Bean, the trip should be free of rustlers, hooligans, or vicious varmints of unsavory kind. Moreover, the speed limit was recently increased to sev-enty miles per hour on most of the road leading to Galveston, so Defendant should be able to hurtle to justice at lightning speed. To assuage Defendant’s worries about the inconvenience of the drive, the Court notes that Houston’s Hobby Airport is located about equal drivetime from downtown Houston and the Galveston courthouse. Defendant will likely find it an easy, traffic-free ride to Galveston as compared to a congested, construction-riddled drive to downtown Houston. The Court notes that any inconvenience suffered in having to drive to Galveston may likely be offset by the peacefulness of the ride and the scenic beauty of the sunny isle.

The convenience of the witnesses and the parties is generally a primary concern of this Court when consider-ing transfer motions. However, vague statements about the convenience of unknown and unnamed witnesses is insufficient to convince this Court that the convenience of the witnesses and the parties would be best served by transferring venue. In the Court’s view, even if all the witnesses, documents, and evidence relevant to this case were located within walking distance of the Houston Division courthouse, the inconvenience caused by retaining the case in this Court would be minimal at best in this age of convenient travel,

communication, discovery, and trial testimony preservation. The Galveston Division courthouse is only about fifty miles from the Houston Division courthouse. “[I]t is

Take Note!

Although the plaintiff has the burden of showing that venue is proper in the face of a challenge to venue, when a defendant seeks a change of venue, that defendant bears the burden of demonstrating that the court should transfer the case. Why do you think the burden is allocated in this way?

Food for Thought

The court makes the argument that modern innovations that make travel and communications more convenient undermine the defendants’ arguments regarding the inconvenience of travel-ing to Galveston rather than Houston. Is this a valid point? Couldn’t one make the same argument about travel-ing to anywhere in the country, that as long as it only takes a flight and a cab ride to get there, convenience is not an issue? At what point does travel become sufficiently inconvenient to warrant a transfer in this “age of con-venient travel”?

Civil Procedure A Contemporary Approach312

not as if the key witnesses will be asked to travel to the wilds of Alaska or the furthest reaches on the Continental United States.”

As to Defendant’s argument that Houston might also be a more convenient forum for Plaintiff, the Court notes that Plaintiff picked Galveston as her forum of choice even though she resides in San Antonio. Defendant argues that flight travel is available between Houston and San Antonio but is not available between Galves-ton and San Antonio, again because of the absence of a commercial airport. Alas, this Court’s kingdom for a commercial airport! The Court is unpersuaded by this argument because it is not this Court’s concern how Plaintiff gets here, whether it be by plane, train, automobile, horseback, foot, or on the back of a huge Texas jackrabbit, as long as Plaintiff is here at the proper date and time. Thus, the Court declines to disturb the forum chosen by the Plaintiff and introduce the likelihood of delay inherent in any transfer simply to avoid the insignificant inconvenience that Defendant may suffer by litigating this matter in Galveston rather than Houston.

For the reasons stated above, Defendant’s Motion to Transfer is hereby DENIED.

_______________

Bolivia v. Philip Morris Companies, Inc.

U.S. District Court for the Southern District of Texas 39 F. Supp. 2d 1008 (S.D. Tex. 1999)

KENT, District Judge.

Plaintiff, the Republic of Bolivia, brings this action to recover from numerous tobacco companies various health care costs it allegedly incurred in treating illnesses its residents suffered as a result of tobacco use. This action was originally filed in the District Court of Brazoria County, Texas * * * and removed to this Court on February 19, 1999, by certain Defen-dants alleging jurisdiction under 28 U.S.C. § 1331 and 28 U.S.C. § 1332. For the following reasons, the Court exercises its authority and discretion pursuant to 28 U.S.C. § 1404(a) to sua sponte TRANSFER this case to the United States District Court for the District of Columbia.

This is one of at least six similar actions brought by foreign governments in vari-ous courts throughout the United States. The governments of Guatemala, Panama,

Take Note!

The judge in this case, Judge Kent is the same judge that decided the previous case, Smith v. Colonial Penn Ins. Co. Ask yourself whether Judge Kent’s determination in this case is consistent with his resolution of the previous case.

Chapter 4 Venue 313

Nicaragua, Thailand, Venezuela, and Bolivia have filed suit in the geographically diverse locales of Washington, D.C., Puerto Rico, Texas, Louisiana, and Florida, in both state and federal courts. Why none of these countries seems to have a court sys-tem their own governments have confidence in is a mystery to this Court. Moreover, given the tremendous number of United States jurisdictions encompassing fascinating and exotic places, the Court can hardly imagine why the Republic of Bolivia elected to file suit in the veritable hinterlands of Brazoria County, Texas. The Court seriously doubts whether Brazoria County has ever seen a live Bolivian . . . even on the Discov-ery Channel. Though only here by removal, this humble Court by the sea is certainly flattered by what must be the worldwide renown of rural Texas courts for dispensing justice with unparalleled fairness and alacrity, apparently in common discussion even on the mountain peaks of Bolivia! Still, the Court would be remiss in accepting an obligation for which it truly does not have the necessary resources. Only one judge presides in the Galveston Division—which currently has before it over seven hundred cases and annual civil filings exceeding such number—and that judge is presently burdened with a significant personal situation which diminishes its ability to always give the attention it would like to all of its daunting docket obligations, despite genu-inely heroic efforts to do so. And, while Galveston is indeed an international seaport, the capacity of this Court to address the complex and sophisticated issues of interna-tional law and foreign relations presented by this case is dwarfed by that of its esteemed colleagues in the District of Columbia who deftly address such awesome tasks as a matter of course. Indeed, this Court, while doing its very best to address the more prosaic matters routinely before it, cannot think of a Bench better versed and more capable of handling precisely this type of case, which requires a high level of expertise in international matters. In fact, proceedings brought by the Republic of Guatemala are currently well underway in that Court in a related action, and there is a request now before the Judicial Panel on Multidistrict Litigation to transfer to the United States District Court for the District of Columbia all six tobacco actions brought by foreign govern-ments, ostensibly for consolidated treatment. Such a Bench, well-populat-ed with genuinely renowned intellects, can certainly better bear and share the burden of multidistrict litigation than this single judge division, where the judge moves his lips when he reads. . . .

* * * [I]t is the Court’s opinion that the District of Columbia, located in this Nation’s capital, is a much more logical venue for the parties and witnesses in this action because, among other things, Plaintiff has an embassy in Washington, D.C., and thus a physical presence and governmental representatives there, whereas there isn’t even a Bolivian restaurant anywhere near here! * * * [T]he Court is virtually

Food for Thought

Should the size of the court’s docket or the judge’s personal problems be legitimate grounds for transferring a case away from a proper venue se-lected by the plaintiff?

Civil Procedure A Contemporary Approach314

certain that Bolivia is not within the four counties over which this Court presides, even though the words Bolivia and Brazoria are a lot alike and caused some real, ini-tial confusion until the Court conferred with its law clerks. Thus, it is readily appar-ent, even from an outdated globe such as that possessed by this Court, that Bolivia, a hemisphere away, ain’t in south-central Texas, and that, at the very least, the District of Columbia is a more appropriate venue (though Bolivia isn’t located there either). Furthermore, as this Judicial District bears no significant relationship to any of the matters at issue, and the judge of this Court simply loves cigars, the Plaintiff can be expected to suffer neither harm nor prejudice by a transfer to Washington, D.C., a Bench better able to rise to the smoky challenges presented by this case, despite the alleged and historic presence there of countless “smoke-filled” rooms. Consequently, pursuant to 28 U.S.C. § 1404(a), for the convenience of parties and witnesses, and in the interest of justice, this case is hereby TRANSFERRED to the United States District Court for the District of Columbia.

_______________

Points for Discussion

a. Standards for Deciding Whether to Transfer

What standards did the court use to evaluate whether a venue transfer was appropriate? Where do you think the court got these considerations from since they are not in the statute? Sec-tion 1404 permits transfers whenever it is “in the interest of justice” to do so, consistent with convenience to the parties and witnesses; is such a stan-dard too indeterminate? See Perspective & Analysis below. Do you think that Judge Kent was justified in denying a transfer in Smith but forcing a transfer, sua sponte, in Philip Morris? These cases involved transfers under §  1404(a). Should the standards that apply to transfers under § 1406(a) be the same? See Sinclair v. Kleindienst, 711 F.2d 291, 294 (D.C. Cir. 1983) (finding it “in the interests of justice” to allow transfer

When a § 1404 transfer is sought pursuant to a forum-selection clause, the Supreme Court has in-structed that the clause is to be given controlling weight, meaning the plaintiff’s choice of forum is given no weight and the convenience interests of the parties and witnesses is simi-larly ignored. Atlantic Marine Const. Co., Inc. v. U.S. Dist. Court for Western Dist. of Texas, 134 S. Ct. 568, 581–82 (2013) [FLEX Case® 4.A]. Transfers under these circumstances can only be defeated if there are public-interest factors such as court congestion or a special need to have localized con-troversies decided at home in courts familiar with the applicable law. Id. at 581 n.6.

FYIFYI

FYI FYI

FYIFYI

Chapter 4 Venue 315

under § 1406(a) where cause of action would be barred by statute of limitations with-out a transfer, personal jurisdiction could be obtained over defendants, and venue is proper in transferee court); see also Loeb v. Bank of Am., 254 F. Supp. 2d 581, 588 (E.D. Pa. 2003) (“[T]he standard for transfer pursuant to § 1404(a) is lower than the standard for transfer pursuant to § 1406(a).”). See 14D C. wright & A. miller, Fed. PrAC. & ProC. 3d § 3808 for a fuller discussion of the factors courts use to evaluate whether a transfer is in the interest of justice.

b. “Divisions” within Districts

Note that in Smith the issue was whether the court would transfer the case from the “Galveston Division” to the “Houston Division” of the United States District Court for the Southern District of Texas. Divisions are simply administrative sub-units within federal districts divided by geographical location. Both of the federal venue transfer provisions apply to transfers to other divisions within a district and the same standards governing whether to grant a transfer to another district apply.

c. Why Allow Transfers at All?

Shouldn’t plaintiffs be entitled to bring and keep their case in the forum of their choice if it is a proper venue under the applicable venue statute? See In re Nat’l Presto Indus., Inc., 347 F.3d 662, 662 (7th Cir. 2003). If an action is brought in a jurisdic-tion having personal jurisdiction over all defendants, why should courts permit a change of venue?

– Perspective & Analysis –

Scholars have criticized the “in the interest of justice” standard of the venue transfer statute as so open-ended that it fosters unpredictability and delay in the handling of motions to transfer:

[T]he law governing section 1404 motions . . . is in chaos. The ad hoc balancing employed by the district courts precludes any prediction on whether a particular transfer motion will succeed. This lack of standards invites a defendant to seek a transfer even when the plaintiff has chosen a reasonable forum. Today, the motion to transfer often has little relationship to the “interests of justice.” Instead, such motions have become a vehicle for defendant delay and an increasing burden on the already backlogged federal courts.

David E. Steinberg, The Motion to Transfer and the Interests of Justice, 66 noTrE damE l. rEv. 443, 446–47 (1990).

Civil Procedure A Contemporary Approach316

d. What Law Applies after a Transfer?

When diversity actions are litigated in federal court, the court follows the choice-of-law rules for the state in which it sits to determine which state’s law to apply. Once a case is transferred to a different federal district court in another state pursuant to § 1404(a), should the transferee court apply the law that would have applied had the case remained with the transferor court or is it free to apply the choice-of-law rules applicable within the transferee court’s state? The Supreme Court has stated that state-law claims transferred under § 1404(a) are to be gov-erned by the same law that would have been applicable in the transferor courts. See Ferens v. John Deere Co., 494 U.S. 516 (1990); Van Dusen v. Barrack, 376 U.S. 612 (1964). See also Note, Choice of Law in Federal Court After Transfer of Venue, 63 Cornell l. rev. 149, 156 (1977). However, if the § 1404 trans-fer was made pursuant to a forum-selection clause, the Van Dusen rule will not apply, meaning the law that would have applied in the transferor court will not follow the case to the transferee district identified in the forum-selection clause. See Atl. Marine Constr. Co. v. U.S. Dist. Court for W. Dist. of Tex., 134 S. Ct. 568, 582 (2013) [FLEX Case® 4.A] (“[W]hen a party bound by a forum-selection clause flouts its contractual obligation and files suit in a different forum, a §1404(a) transfer of venue will not carry with it the original venue’s choice-of-law rules.”).

In federal question cases, courts are to apply governing federal law, which can vary among the district courts depending upon which federal circuit embraces their district. When a case is transferred under § 1404(a) from a district in one circuit to another district in a different circuit, which circuit’s law should the transferee court apply? See In re Korean Air Lines Disaster of Sept. 1, 1983, 829 F.2d 1171 (D.C. Cir. 1987) (indicating that transferee court is not bound by transferor circuit law).

_______________

C. Forum Non Conveniens

As we have just learned, the federal change of venue provisions only permit the transfer of a case to another federal court. However, it may sometimes be the case that the forum that would be more just or convenient is not within the federal sys-tem. What mechanism is available to federal courts once they have determined that a case before them would be better located in a court outside of the federal system,

When a court has to resolve the substance of a dispute based onstate law, it first has to determine which state’s substantive law to ap-ply. Courts make this determina-tion by reference to the forum state’s choice-of-law rules.

FYIFYI

FYI FYI

FYIFYI

Chapter 4 Venue 317

such as a state court or a court in a foreign country? The next case involves the doc-trine of forum non conveniens, a common law (judicially-created or non-statutory) doctrine for handling cases that would be better tried in a different judicial system.

Piper Aircraft Co. v. Reyno

Supreme Court of the United States 454 U.S. 235 (1981)

Justice MARSHALL delivered the opinion of the Court.

These cases arise out of an air crash that took place in Scotland. Respondent, acting as representative of the estates of several Scottish citizens killed in the acci-dent, brought wrongful-death actions against petitioners that were ultimately transferred to the United States Dis-trict Court for the Middle District of Pennsylvania. Petitioners moved to dismiss on the ground of forum non conveniens. After noting that an alter-native forum existed in Scotland, the District Court granted their motions. The United States Court of Appeals for the Third Circuit reversed. The Court of Appeals based its decision, at least in part, on the ground that dismissal is automatically barred where the law of the alternative forum is less favorable to the plaintiff than the law of the forum chosen by the plaintiff. * * *

I

A

In July 1976, a small commercial aircraft crashed in the Scottish highlands during the course of a charter flight from Blackpool to Perth. The pilot and five pas-sengers were killed instantly. The decedents were all Scottish subjects and residents, as are their heirs and next of kin. There were no eyewitnesses to the accident. At the time of the crash the plane was subject to Scottish air traffic control.

The aircraft, a twin-engine Piper Aztec, was manufactured in Pennsylvania by petitioner Piper Aircraft Co. (Piper). The propellers were manufactured in Ohio by petitioner Hartzell Propeller, Inc. (Hartzell). At the time of the crash the aircraft was registered in Great Britain and was owned and maintained by Air Navigation and Trading Co., Ltd. (Air Navigation). It was operated by McDonald Aviation, Ltd.

Make the Connection

Wrongful-death actions are suits by the survivors of a person who has been killed (or by the representa-tive of the decedent’s estate) that seek compensation for the loss of that person’s life. You may study this cause of action further in your first-year Torts course.

Civil Procedure A Contemporary Approach318

(McDonald), a Scottish air taxi service. Both Air Navigation and McDonald were organized in the United Kingdom. The wreckage of the plane is now in a hangar in Farnsborough, England.

The British Department of Trade investigated the accident shortly after it occurred. A preliminary report found that the plane crashed after developing a spin, and sug-gested that mechanical failure in the plane or the propeller was responsible. At Hartz-ell’s request, this report was reviewed by a three-member Review Board, which held a 9-day adversary hearing attended by all interested parties. The Review Board found no evidence of defective equipment and indicated that pilot error may have contributed to the accident. The pilot, who had obtained his commercial pilot’s license only three months earlier, was flying over high ground at an altitude considerably lower than the minimum height required by his company’s operations manual.

In July 1977, a California probate court appointed respondent Gaynell Reyno administratrix of the estates of the five passengers. Reyno is not related to and does not know any of the decedents or their survivors; she was a legal secretary to the attorney who filed this lawsuit. Several days after her appointment, Reyno com-menced separate wrongful-death actions against Piper and Hartzell in the Superior Court of California, claiming negligence and strict liability. Air Navigation, McDon-ald, and the estate of the pilot are not parties to this litigation. The survivors of the five passengers whose estates are represented by Reyno filed a separate action in the United Kingdom against Air Navigation, McDonald, and the pilot’s estate. Reyno candidly admits that the action against Piper and Hartzell was filed in the United

States because its laws regarding liability, capacity to sue, and damages are more favorable to her position than are those of Scotland. Scottish law does not recog-nize strict liability in tort. Moreover, it permits wrongful-death actions only when brought by a decedent’s relatives. The relatives may sue only for “loss of support and society.”

On petitioners’ motion, the suit was removed to the United States District Court for the Central District of California. Piper then moved for transfer to the United States District Court for the Middle District of Pennsylvania, pursuant to 28 U.S.C. §1404(a). Hartzell moved to dismiss for lack of personal jurisdiction, or in the

Food for Thought

Why is the attorney’s legal secretary being used as the administratrix in this case? Does this arrangement raise any concerns for you?

Make the Connection

Strict liability is a Tort concept that refers to the rule that permits liability to be established based solely upon a showing of causation, without hav-ing to establish fault or culpability.

Chapter 4 Venue 319

alternative, to transfer.5 In December 1977, the District Court quashed service on Hartzell and transferred the case to the Middle District of Pennsylvania. Respondent then properly served process on Hartzell.

B

In May 1978, after the suit had been transferred, both Hartzell and Piper moved to dismiss the action on the ground of forum non conveniens. The District Court granted these motions in October 1979. It relied on the balancing test set forth by this Court in Gulf Oil Corp. v. Gilbert, 330 U.S. 501 (1947) * * * . In those decisions, the Court stated that a plaintiff’s choice of forum should rarely be disturbed. How-ever, when an alternative forum has jurisdiction to hear the case, and when trial in the chosen forum would “establish . . . oppressiveness and vexation to a defendant . . . out of all proportion to plaintiff’s convenience,” or when the “chosen forum [is] inappropriate because of considerations affecting the court’s own administrative and legal problems,” the court may, in the exercise of its sound discretion, dismiss the case. To guide trial court discretion, the Court provided a list of “private interest fac-tors” affecting the convenience of the litigants, and a list of “public interest factors” affecting the convenience of the forum. Gilbert, 330 U.S. at 508–09.6

* * * [T]he District Court analyzed the facts of [this case]. It began by observ-ing that an alternative forum existed in Scotland; Piper and Hartzell had agreed to submit to the jurisdiction of the Scottish courts and to waive any statute of limita-tions defense that might be available. It then stated that plaintiff’s choice of forum was entitled to little weight. The court recognized that a plaintiff’s choice ordinarily deserves substantial deference. It noted, however, that Reyno “is a representative of foreign citizens and residents seeking a forum in the United States because of the more liberal rules concerning products liability law,” and that “the courts have been less solicitous when the plaintiff is not an American citizen or resident, and particularly when the foreign citizens seek to benefit from the more liberal tort rules provided for the protection of citizens and residents of the United States.”

The District Court next examined several factors relating to the private interests of the litigants, and determined that these factors strongly pointed towards Scotland

5 The District Court concluded that it could not assert personal jurisdiction over Hartzell consistent with due process. However, it decided not to dismiss Hartzell because the corporation would be amenable to process in Pennsylvania.6 The factors pertaining to the private interests of the litigants included the “relative ease of access to sources of proof; availability of compulsory process for attendance of unwilling, and the cost of obtaining attendance of willing, witnesses; possibility of view of premises, if view would be appropriate to the action; and all other practical problems that make trial of a case easy, expeditious and inexpensive.” Gilbert, 330 U.S. at 508. The public factors bearing on the question included the administrative difficul-ties flowing from court congestion; the “local interest in having localized controversies decided at home”; the interest in having the trial of a diversity case in a forum that is at home with the law that must govern the action; the avoidance of unnecessary problems in conflict of laws, or in the application of foreign law; and the unfairness of burdening citizens in an unrelated forum with jury duty. Id. at 509.

Civil Procedure A Contemporary Approach320

as the appropriate forum. Although evidence concerning the design, manufacture, and testing of the plane and propeller is located in the United States, the connec-tions with Scotland are otherwise “overwhelming.” The real parties in interest are citizens of Scotland, as were all the decedents. Witnesses who could testify regarding the maintenance of the aircraft, the training of the pilot, and the investigation of the accident-all essential to the defense-are in Great Britain. Moreover, all witnesses to damages are located in Scotland. Trial would be aided by familiarity with Scottish topography, and by easy access to the wreckage.

The District Court reasoned that because crucial witnesses and evidence were beyond the reach of compulsory process, and because the defendants would not be able to implead potential Scottish third-party defendants, it would be “unfair to make Piper and Hartzell proceed to trial in this forum.” The survivors had brought separate actions in Scotland against the pilot, McDonald, and Air Navigation. “[I]t would be fairer to all parties and less costly if the entire case was presented to one jury with available testimony from all relevant witnesses.” Although the court recognized that if trial were held in the United States, Piper and Hartzell could file indemnity or con-tribution actions against the Scottish defendants, it believed that there was a significant risk of inconsistent ver-dicts.

The District Court concluded that the relevant public interests also pointed strongly towards dismissal. The court determined that Pennsylvania law would apply to Piper and Scottish law to Hartzell if the case were tried in the Middle District of Pennsylvania.8 As a result, “trial in this forum would be hopelessly com-plex and confusing for a jury.” In addition, the court noted that it was unfamiliar with Scottish law and thus would have to rely upon experts from that country. The court also found that the trial would be enormously costly and time-consuming; that it would be unfair to burden citizens with jury duty when the Middle District of Pennsylvania has little connection with the controversy; and that Scotland has a substantial interest in the outcome of the litigation.

8 Under Klaxon v. Stentor Electric Mfg. Co., 313 U.S. 487 (1941), a court ordinarily must apply the choice-of-law rules of the State in which it sits. However, where a case is transferred pursuant to 28 U.S.C. § 1404(a), it must apply the choice-of-law rules of the State from which the case was transferred. Van Dusen v. Barrack, 376 U.S. 612 (1964). Relying on these two cases, the District Court concluded that California choice-of-law rules would apply to Piper, and Pennsylvania choice-of-law rules would apply to Hartzell. It further concluded that California applied a “governmental interests” analysis in resolving choice-of-law problems, and that Pennsylvania employed a “significant contacts” analysis. The court used the “governmental interests” analysis to determine that Pennsylvania liability rules would apply to Piper, and the “significant contacts” analysis to determine that Scottish liability rules would apply to Hartzell.

What’s That?

The term implead means to bring a nonparty into the action (thereby making it a third-party defendant) under Rule 14, which will be cov-ered in Chapter 7.

???

?

?

?

Chapter 4 Venue 321

In opposing the motions to dismiss, respondent contended that dismissal would be unfair because Scottish law was less favorable. The District Court explic-itly rejected this claim. It reasoned that the possibility that dismissal might lead to an unfavorable change in the law did not deserve significant weight; any deficiency in the foreign law was a “matter to be dealt with in the foreign forum.”

C

On appeal, the United States Court of Appeals for the Third Circuit reversed and remanded for trial. The decision to reverse appears to be based on two alter-native grounds. First, the Court held that the District Court abused its discretion in conducting the Gilbert analysis. Second, the Court held that dismissal is never appropriate where the law of the alternative forum is less favorable to the plaintiff.

The Court of Appeals began its review of the District Court’s Gilbert analysis by noting that the plaintiff’s choice of forum deserved substantial weight, even though the real parties in interest are nonresidents. It then rejected the District Court’s bal-ancing of the private interests. It found that Piper and Hartzell had failed adequately to support their claim that key witnesses would be unavailable if trial were held in the United States: they had never specified the witnesses they would call and the testimony these witnesses would provide. The Court of Appeals gave little weight to the fact that Piper and Hartzell would not be able to implead potential Scottish third-party defendants, reasoning that this difficulty would be “burdensome” but not “unfair.” Finally, the court stated that resolution of the suit would not be signifi-cantly aided by familiarity with Scottish topography, or by viewing the wreckage.

The Court of Appeals also rejected the District Court’s analysis of the public interest factors. It found that the District Court gave undue emphasis to the applica-tion of Scottish law: “the mere fact that the court is called upon to determine and apply foreign law does not present a legal problem of the sort which would justify the dismissal of a case otherwise properly before the court.” In any event, it believed that Scottish law need not be applied. After conducting its own choice-of-law analy-sis, the Court of Appeals determined that American law would govern the actions against both Piper and Hartzell. The same choice-of-law analysis apparently led it to conclude that Pennsylvania and Ohio, rather than Scotland, are the jurisdictions with the greatest policy interests in the dispute, and that all other public interest factors favored trial in the United States.

The Court of Appeals concluded as part of its choice-of-law analysis that the United States had the greatest policy interest in the dispute. It apparently believed that this conclusion necessarily implied that the forum non conveniens public interest factors pointed toward trial in the United States.

In any event, it appears that the Court of Appeals would have reversed even if the District Court had properly balanced the public and private interests. The court stated:

Civil Procedure A Contemporary Approach322

[I]t is apparent that the dismissal would work a change in the applicable law so that the plaintiff’s strict liability claim would be eliminated from the case. But . . . a dismissal for forum non conveniens, like a statutory transfer, “should not, despite its convenience, result in a change in the applicable law.” Only when American law is not applicable, or when the foreign jurisdiction would, as a matter of its own choice of law, give the plaintiff the benefit of the claim to which she is entitled here, would dismissal be justified.

In other words, the court decided that dismissal is automatically barred if it would lead to a change in the applicable law unfavorable to the plaintiff.

* * *

II

The Court of Appeals erred in holding that plaintiffs may defeat a motion to dismiss on the ground of forum non conveniens merely by showing that the sub-stantive law that would be applied in the alternative forum is less favorable to the plaintiffs than that of the present forum. The possibility of a change in substantive law should ordinarily not be given conclusive or even substantial weight in the forum non conveniens inquiry.

* * *

In fact, if conclusive or substantial weight were given to the possibility of a change in law, the forum non conveniens doctrine would become virtually useless. Jurisdiction and venue requirements are often easily satisfied. As a result, many plaintiffs are able to choose from among several forums. Ordinarily, these plaintiffs will select that forum whose choice-of-law rules are most advantageous. Thus, if the possibility of an unfavorable change in substantive law is given substantial weight in the forum non conveniens inquiry, dismissal would rarely be proper.

* * *

The Court of Appeals’ approach is not only inconsistent with the purpose of the forum non conveniens doctrine, but also poses substantial practical problems. If the possibility of a change in law were given substantial weight, deciding motions to dismiss on the ground of forum non conveniens would become quite difficult. Choice-of-law analysis would become extremely important, and the courts would frequently be required to interpret the law of foreign jurisdictions. First, the trial court would have to determine what law would apply if the case were tried in the chosen forum, and what law would apply if the case were tried in the alternative forum. It would then have to compare the rights, remedies, and procedures available under the law that would be applied in each forum. Dismissal would be appropriate only if the court concluded that the law applied by the alternative forum is as favorable to the plaintiff as that of the chosen forum. The doctrine of forum non conveniens, however,

Chapter 4 Venue 323

is designed in part to help courts avoid conducting complex exercises in compara-tive law. As we stated in Gilbert, the public interest factors point towards dismissal where the court would be required to “untangle problems in conflict of laws, and in law foreign to itself.”

Upholding the decision of the Court of Appeals would result in other practical problems. At least where the foreign plaintiff named an American manufacturer as defendant, a court could not dismiss the case on grounds of forum non conveniens where dismissal might lead to an unfavorable change in law. The American courts, which are already extremely attractive to foreign plaintiffs,18 would become even more attractive. The flow of litigation into the United States would increase and further congest already crowded courts. * * *

We do not hold that the possibil-ity of an unfavorable change in law should never be a relevant consider-ation in a forum non conveniens inquiry. Of course, if the remedy provided by the alternative forum is so clearly inadequate or unsatisfactory that it is no remedy at all, the unfavorable change in law may be given substantial weight; the district court may con-clude that dismissal would not be in the interests of justice.22 In these cases, however, the remedies that would be provided by the Scottish courts do not fall within this category. Although the relatives of the decedents may not be able to rely on a strict liability theory, and although their potential damages award may be smaller, there is no dan-ger that they will be deprived of any remedy or treated unfairly.

18 First, all but 6 of the 50 American States—Delaware, Massachusetts, Michigan, North Carolina, Vir-ginia, and Wyoming—offer strict liability. * * * [S]trict liability remains primarily an American innova-tion. Second, the tort plaintiff may choose, at least potentially, from among 50 jurisdictions if he decides to file suit in the United States. Each of these jurisdictions applies its own set of malleable choice-of-law rules. Third, jury trials are almost always available in the United States, while they are never provided in civil law jurisdictions. * * * Fourth, unlike most foreign jurisdictions, American courts allow contingent attorney’s fees, and do not tax losing parties with their opponents’ attorney’s fees. Fifth, discovery is more extensive in American than in foreign courts.22 At the outset of any forum non conveniens inquiry, the court must determine whether there exists an alternative forum. Ordinarily, this requirement will be satisfied when the defendant is “amenable to process” in the other jurisdiction. Gilbert, 330 U.S. at 506–507. In rare circumstances, however, where the remedy offered by the other forum is clearly unsatisfactory, the other forum may not be an adequate alternative, and the initial requirement may not be satisfied. Thus, for example, dismissal would not be appropriate where the alternative forum does not permit litigation of the subject matter of the dispute.

Take Note!

The Court here indicates that the district court did not abuse its discre-tion. Make note that this is a reference to the standard of review applicable here: the district court’s application of the Gilbert factors is entitled to defer-ence and may only be overturned by appellate courts if the district court abused its discretion. See Part III.B of the Opinion below. Contrast this with the de novo standard of review, which permits appellate courts to make their own independent analysis of the mat-ter at hand without giving any defer-ence to the district court’s analysis. Why do you think an abuse of dis-cretion standard is appropriate for the review of a district court’s application of the Gilbert factors? Standards of review will be covered more fully in Chapter 11.

Civil Procedure A Contemporary Approach324

III

The Court of Appeals also erred in rejecting the District Court’s Gilbert analysis. The Court of Appeals stated that more weight should have been given to the plain-tiff’s choice of forum, and criticized the District Court’s analysis of the private and public interests. However, the District Court’s decision regarding the deference due plaintiff’s choice of forum was appropriate. Furthermore, we do not believe that the District Court abused its discretion in weighing the private and public interests.

A

The District Court acknowledged that there is ordinarily a strong presump-tion in favor of the plaintiff’s choice of forum, which may be overcome only when the private and public interest factors clearly point towards trial in the alternative forum. It held, however, that the presumption applies with less force when the plaintiff or real parties in interest are foreign.

The District Court’s distinction between resident or citizen plaintiffs and foreign plaintiffs is fully justified. * * * When the home forum has been chosen, it is reason-able to assume that this choice is convenient. When the plaintiff is foreign, however, this assumption is much less reasonable. Because the central purpose of any forum non conveniens inquiry is to ensure that the trial is convenient, a foreign plaintiff’s choice deserves less deference.

B

The forum non conveniens determination is committed to the sound discretion of the trial court. It may be reversed only when there has been a clear abuse of discretion; where the court has considered all relevant public and private interest factors, and where its balancing of these factors is reasonable, its decision deserves substantial deference. Here, the Court of Appeals expressly acknowledged that the standard of review was one of abuse of discretion. In examining the District Court’s analysis of the public and private interests, however, the Court of Appeals seems to have lost sight of this rule, and substituted its own judgment for that of the District Court.

(1)

In analyzing the private interest factors, the District Court stated that the con-nections with Scotland are “overwhelming.” This characterization may be some-what exaggerated. Particularly with respect to the question of relative ease of access to sources of proof, the private interests point in both directions. As respondent emphasizes, records concerning the design, manufacture, and testing of the propel-ler and plane are located in the United States. She would have greater access to

Chapter 4 Venue 325

sources of proof relevant to her strict liability and negligence theories if trial were held here. However, the District Court did not act unreasonably in concluding that fewer evidentiary problems would be posed if the trial were held in Scotland. A large proportion of the relevant evidence is located in Great Britain.

The Court of Appeals found that the problems of proof could not be given any weight because Piper and Hartzell failed to describe with specificity the evidence they would not be able to obtain if trial were held in the United States. It sug-gested that defendants seeking forum non conveniens dismissal must submit affidavits identifying the witnesses they would call and the testimony these witnesses would provide if the trial were held in the alternative forum. Such detail is not necessary. Piper and Hartzell have moved for dismissal precisely because many crucial wit-nesses are located beyond the reach of compulsory process, and thus are difficult to identify or interview. Requiring extensive investigation would defeat the purpose of their motion. Of course, defendants must provide enough information to enable the District Court to balance the parties’ interests. Our examination of the record convinces us that sufficient information was provided here. Both Piper and Hartzell submitted affidavits describing the evidentiary problems they would face if the trial were held in the United States.

The District Court correctly concluded that the problems posed by the inability to implead potential third-party defendants clearly supported holding the trial in Scotland. Joinder of the pilot’s estate, Air Navigation, and McDonald is crucial to the presentation of petitioners’ defense. If Piper and Hartzell can show that the accident was caused not by a design defect, but rather by the negligence of the pilot, the plane’s owners, or the charter company, they will be relieved of all liability. It is true, of course, that if Hartzell and Piper were found liable after a trial in the United States, they could institute an action for indemnity or contribution against these parties in Scotland. It would be far more convenient, however, to resolve all claims in one trial. The Court of Appeals rejected this argument. Forcing petitioners to rely on actions for indemnity or contributions would be “burdensome” but not “unfair.” Finding that trial in the plaintiff’s chosen forum would be burdensome, however, is sufficient to support dismissal on grounds of forum non conveniens.

(2)

The District Court’s review of the factors relating to the public interest was also reasonable. On the basis of its choice-of-law analysis, it concluded that if the case were tried in the Middle District of Pennsylvania, Pennsylvania law would apply to Piper and Scottish law to Hartzell. It stated that a trial involving two sets of laws would be confusing to the jury. It also noted its own lack of familiarity with Scottish law. Consideration of these problems was clearly appropriate under Gilbert; in that case we explicitly held that the need to apply foreign law pointed towards dismissal. The Court of Appeals found that the District Court’s choice-of-law analysis was

Civil Procedure A Contemporary Approach326

incorrect, and that American law would apply to both Hartzell and Piper. Thus, lack of familiarity with foreign law would not be a problem. Even if the Court of Appeals’ conclusion is correct, however, all other public interest factors favored trial in Scotland.

Scotland has a very strong interest in this litigation. The accident occurred in its airspace. All of the decedents were Scottish. Apart from Piper and Hartzell, all potential plaintiffs and defendants are either Scottish or English. As we stated in Gil-bert, there is “a local interest in having localized controversies decided at home.” 330 U.S. at 509. Respondent argues that American citizens have an interest in ensuring that American manufacturers are deterred from producing defective products, and that additional deterrence might be obtained if Piper and Hartzell were tried in the United States, where they could be sued on the basis of both negligence and strict liability. However, the incremental deterrence that would be gained if this trial were held in an American court is likely to be insignificant. The American interest in this accident is simply not sufficient to justify the enormous commitment of judicial time and resources that would inevitably be required if the case were to be tried here.

IV

The Court of Appeals erred in holding that the possibility of an unfavorable change in law bars dismissal on the ground of forum non conveniens. It also erred in rejecting the District Court’s Gilbert analysis. The District Court properly decided that the presumption in favor of the respondent’s forum choice applied with less than maximum force because the real parties in interest are foreign. It did not act unreasonably in deciding that the private interests pointed towards trial in Scotland. Nor did it act unreasonably in deciding that the public interests favored trial in Scotland. Thus, the judgment of the Court of Appeals is

Reversed.

Justice POWELL took no part in the decision of these cases.

Justice O’CONNOR took no part in the consideration or decision of these cases.

[The concurring opinion of Justice WHITE and the dissenting opinion of Justice STEVENS joined by Justice BRENNAN are omitted.]

_______________

Page 305

Insert 1:

Page 306

Insert 1:

Page 318

Insert 1:

Page 319

Insert 1:

Chapter 4 Venue 327

Points for Discussion

a. Standards for Deciding Whether to Dismiss for Forum Non Conveniens

Based on what you have read in Piper Aircraft, what are the requirements for dismissing a case on forum non conveniens grounds? What role do the factors set forth by the Court in Gulf Oil Corp. v. Gilbert, 330 U.S. 501 (1947), and quoted by the Piper Aircraft Court in footnote 6 above play in the analysis?

b. An Adequate Alternative Forum

An important element of granting a forum non conveniens dismissal is that there is an adequate alternate forum where the plaintiff may proceed with the case. What issues are relevant to determining whether an adequate alternate forum exists? In Piper Aircraft the Court rejected using the fact that less favorable substantive law would apply as a basis for denying a forum non conveniens dismissal. Do you agree with that decision?

Consider whether the decision in this case would have changed if Scotland did not permit recovery for wrongful death under these circumstances or if its legal system failed to provide basic due process protections available in the United States. Alternatively, what if the damages for wrongful death in Scotland were capped at an unusually low figure, such as $10,000 per person; would Scotland still be viewed as an adequate alternate forum? See Gonzalez v. Chrysler Corp., 301 F.3d 377, 382–83 (5th Cir. 2002) (holding that the adequacy analysis for forum non conveniens dismiss-als may not include an evaluation of whether it makes economic sense for the plain-tiff to proceed in an alternate foreign jurisdiction). At what point should differences in substantive law impact the forum non conveniens analysis?

c. Forum Non Conveniens Dismissals Prior to Jurisdictional Determinations

The Supreme Court recently determined that courts need not resolve other threshold objections—such as whether the court has subject matter jurisdiction or personal jurisdiction—prior to addressing a defendant’s forum non conveniens motion and dismissing the case on that basis. Sinochem Int’l Co. v. Malaysia Int’l Shipping Corp., 549 U.S. 422 (2007).

d. After Dismissal

What happens after a court dismisses a case on forum non conveniens grounds? Plaintiffs then have the option to refile their cases in an alternate forum. However, there could be difficulties associated with doing so. For example, the alternate forum may lack personal jurisdiction over the defendant or the statute of limitations period may have run. When there is a possibility or probability that the plaintiff will face such challenges to refiling its case in the wake of a forum non conveniens dismissal,

Civil Procedure A Contemporary Approach328

the dismissing court typically dismisses the case only on the condition that the defendant will not contest the ability of the subsequent court to hear the case. See, e.g., Pollux Holding Ltd. v. Chase Manhattan Bank, 329 F.3d 64 (2d Cir. 2003), cert. denied, 540 U.S. 1149 (2004) (affirming district court’s dismissal of case filed in New York against defendant British bank, when defendant had agreed as a condition to submit to the jurisdiction of British courts).

Do courts have authority to impose such conditions on defendants when grant-ing forum non conveniens dismissals if they have not first determined whether they have subject matter jurisdiction over the claims and personal jurisdiction over the defendants, a determination the Supreme Court in Sinochem said courts do not have to make? The Court avoided this issue in Sinochem itself. See Sinochem, 549 U.S. at 435 (“We . . . need not decide whether a court conditioning a forum non conve-niens dismissal on the waiver of juris-dictional or limitations defenses in the foreign forum must first determine its own authority to adjudicate the case.”).

If federal court dismisses a claim on forum non conveniens grounds in favor of litigating the case in the court of another country, may the plaintiff refile the case in a state court instead? If the plaintiff does so, may the defendant get a federal court injunction prohibiting the state court action as contrary to the previous forum non conveniens dismissal? See Chick Kam Choo v. Exxon Corp., 486 U.S. 140, 146 (1988) (rejecting such an injunction entered after a federal forum non conveniens dismissal and holding that state court was free to conduct its own forum non conveniens analysis because “whether the Texas state courts are an appropriate forum for [the plaintiff’s] Singapore law claims has not yet been litigated”).

e. Forum Non Conveniens and the Erie Doctrine

As you will learn in Chapter 5, under the doctrine of Erie R.R. Co. v. Tompkins, 304 U.S. 64 (1938), federal courts sitting in diversity are obligated to apply the substantive state law that would be applied to a claim in the state where the federal court hearing the case is located. Does this requirement apply to the law of forum non conveniens? In other words, may federal courts follow federal forum non conveni-ens doctrine or must they apply the relevant state approach to forum non conveniens? The Supreme Court has suggested, but not held, that the federal law of forum non conveniens is a procedural doctrine that federal courts must apply but that state courts may ignore. See Am. Dredging Co. v. Miller, 510 U.S. 443, 453 (1994) (stating that “forum non conveniens . . . is procedural rather than substantive”).

_______________

Practice Pointer

As counsel for a plaintiff facing a forum non conveniens dismissal, be sure to pursue and secure conditions of the dismissal from the court that require the defendant to waive all jurisdictional, venue, statute of limi-tations, and other challenges to the ability of a subsequent court to hear your client’s case.

Chapter 4 Venue 329

Executive Summary

n Basic Venue. 28 U.S.C. § 1391 is the general venue statute. Careful appli-cation of its provisions is necessary to determine whether venue is proper in cases where the statute applies.

n Special Venue Statutes. When there are special venue statutes that apply, they generally serve to supplement the general venue statute unless the special stat-ute is exclusive or somehow conflicts with the general venue statute.

n Pendent Venue. Although venue must be proper with respect to each claim, many courts recognize the concept of pendent venue, which will permit proper venue for one claim to establish proper venue for all claims that share a common nucleus of operative fact. This, however, is not a universally accepted principle.

n Change of Venue (Trans-fer). 28 U.S.C. § 1404 permits courts to transfer a properly venued case to another federal district or division in which the case could have been brought initially, if doing so furthers the convenience of the witnesses and parties and is in the interests of justice. 28 U.S.C. §  1406 permits courts to transfer improperly venued cases to other federal districts where they could have been brought if the interests of justice suggest that such a transfer is warranted.

n Applicable Law after Transfer. When state law claims are transferred from proper venues, the law that would have applied in the transferor court applies in the transferee court. When federal question claims are transferred, the transferee court is not bound by the circuit law of the transferor court.

n Forum Non Conveniens. When the analysis of the Gilbert private and pub-lic interest factors indicates that a more appropriate and adequate alternate forum exists for the action, but the alternate forum is within a separate judicial system, a

Major Themes

Keep in mind two overarching themes at work within the area of venue:

a. Forum Shopping—the venue statute may often give plaintiffs an array of proper venues in which to file their actions. Although this de-cision will ultimately be constrained in part by personal jurisdiction con-siderations, plaintiffs here have some ability to identify the forum that is best for their interests from a strate-gic perspective.

b. Defendant Veto—although the defendant does not have an abso-lute “veto” over the forum selection of the plaintiff, the venue transfer provisions along with the doctrine of forum non conveniens do enable the defendant to attempt to convince the court that the action should be heard in a court other than that selected initially by the plaintiff.

Civil Procedure A Contemporary Approach330

court may dismiss the case on forum non conveniens grounds. When doing so, courts typically will condition the dismissal on the defendant agreeing to permit the case to be litigated in the alternate forum, waiving any jurisdictional or other challenges to the alternate court’s authority to adjudicate the dispute.

For More Information

Students interested in obtaining more information about venue may consult the follow-ing resources:

• 14D C. wright, A. miller & e. CooPer, Fed. PrAC. & ProC. Juris. 3d § 3801 et seq.

• Mary Garvey Algero, In Defense of Forum Shopping: A Realistic Look at Selecting a Venue, 78 neb. l. rev. 79 (1999).

• Kimberly Jade Norwood, Shopping for a Venue: The Need for More Limits on Choice, 50 u. miAmi l. rev. 267 (1996).

i